Download Toxicology expert questions General principles of toxicology

Document related concepts

Adherence (medicine) wikipedia , lookup

Management of multiple sclerosis wikipedia , lookup

Transcript
Toxicology expert questions
General principles of toxicology
Management issues in gastrointestinal decontamination





Emesis
Gastric lavage
Activated charcoal
Cathartics
Whole bowel irrigation
Gastrointestinal decontamination triangle
Though historically used following most ingestion with the aim to reduce the load of ingested toxin,
gastrointestinal decontamination measures has not been always beneficial to patients.
Methods of gastrointestinal decontamination include
 Induced emesis
 Gastric lavage
 Activated charcoal
 Whole bowel irrigation
The tendency in the past has been to overestimate the potential benefits while underestimating the potential
hazards of GI decontamination procedures. These procedures do not provide significant benefit when applied
to unselected patients and should no longer be considered routine.
The decision to decontaminate is one of clinical judgment in which the potential benefits outweigh the
potential risks and the resources required to perform the procedure. Thus GI decontamination is reserved for
cases where the risk assessment predicts severe or life threatening toxicity and where supportive care or
antidote treatment alone is insufficient to ensure a satisfactory outcome.
Potential benefits and risks of GI decontamination
Potential benefits
Potential risks
 Improved clinical outcomes (morbidity and
 Pulmonary aspiration
mortality)
 More benign clinical course (requiring lower level
 GI complications
of supportive care)
o Bowel obstruction
o perforation
 reduced need of potentially hazardous
 distraction of staff from resuscitation and
interventions or expensive antidotes
supportive care priorities
 reduced hospital length of stay
 diversion of departmental resources for
performance of procedure
Indications for GI decontamination:
 risk assessment predicts severe or life threatening toxicity and where supportive care or antidote
treatment alone is insufficient to ensure a satisfactory outcome
 there should be reasonable grounds to believe that a significant amount of agent remains unabsorbed
 the ingested agent is amenable to removal by the selected procedure
Contraindications for GI decontamination:
 if the procedure is detrimental to basic resuscitation or supportive care
 if the airway is not secured
 risk assessment predicts the potential for imminent seizures or decline in LOC
Induced Emesis


Achieved almost exclusively by the administration of syrup of Ipecac.
Preparation contains plant-derived emetics which in recommended doses, reliably induce vomiting via
central and peripheral mechanisms
 Mean time from administration to vomiting is 18mins
 Amount of toxin removed varies greatly and reduces over time to almost negligible at 1 hour.
Indications
 When administered promptly after ingestion of agent in a dose likely to cause significant toxicity,
where activated charcoal is not available and not likely to be of benefit
 Very rarely such a scenario occurs in the ED setting and so most EDs do not stock it anymore
Technique
 15ml(Children) or 15-30ml with a glass of water, if vomiting does not occur in 30mins, repeat dose
Contraindications
 Non-toxic ingestion
 Seizures or decreased level of consciousness – present or expected
 Activated charcoal likely to be available within 1 hr and known to work in situation
 Corrosive ingestion
 Hydrocarbon ingestion
Potential complications
 Prolonged vomiting (10-20% vomit >1hr)
 Diarrhea (20-30%)
 Lethargy (10%)
 Pulmonary aspiration
 Physical injuries secondary to vomiting (rare)
o Mallory Weiss tear
o Pneumomediastinum
o Gastric perforation
Gastric Lavage
Gastric lavage involves attempts to empty the stomach of toxic substances by sequential administration and
aspiration of small amounts of fluid from the stomach via an OG tube.
 Amount of toxin removed is unreliable and negligible if performed after the first hour.
 Very few situations where its administration has benefits more than that of administration of
activated charcoal
Absolute contraindications
 Initial resuscitation incomplete
 Antidote therapy available with indication of good outcome with its use
 Unprotected airway
 Small children
 Corrosive or hydrocarbon ingestion
Potential complications
 Pulmonary aspiration
 Hypoxia
 Laryngospasm
 Mechanical injury to GIT
 Water intoxication in children
 Hypothermia
 Resource distraction away from resuscitation and supportive care
Single-dose Activated Charcoal
Activated Charcoal (AC) is produced by the super-heating of distilled wood pulp. The resulting fine porous
particles are suspended in water or sorbitol. The enormous surface area provided by these particles reversibly
adsorbs most ingested toxins preventing further adsorption from the GIT.
Indications
Where it is likely that the toxin remains in the GIT (<1hr for most agents) and benefits outweigh risks.
Complications
 Vomiting (30% within 1hr)
 Messy
 Iatrogenic administration into lungs via misplaced NG tube
 Pulmonary aspiration
 Impaired absorption of subsequently administered oral medications/ antidotes
 Resource distraction away from resuscitation and supportive care
Contraindications
 Initial resuscitation incomplete
 Non-toxic ingestion
 Good outcome predicted with supportive care or antidote therapy
 Unprotected airway
 Agent not bound by AC
 Corrosive ingestion
Agents poorly bound to AC
Hydrocarbons and alcohols
Metals
Corrosives
 Ethanol
 Lithium
 Acids
 Isopropyl alcohol
 Iron
 alkalis
 Ethylene glycol
 Potassium
 Methanol
 Lead
 Arsenic
 Mercury
Technique
 50g in adults and 1g/kg in children as a single dose orally or NG after confirmation of placement.
Whole Bowel Irrigation (WBI)
WBI is a labour intensive form of GI decontamination which attempts to cleanse the entire bowel by
administering large volumes of osmotically-balanced polyethylene glycol-electrolyte solution (PEG-ELS).
Indications



Due to its labor intensiveness, is reserved for life-threatening ingestions of sustained-release or
enteric coated preparations
Agents that do not bind with AC and
Where good clinical outcome is not expected with supportive care and antidote administration and
patient presents before established toxicity
WBI potentially useful
 Iron overdose >60mg/kg
 SR potassium chloride ingestion >2.5mmol/kg
 Life-threatening SR verapamil or diltiazem ingestions
 Lead ingestion
 Body packers
Complications
 Nausea, vomiting and abdominal bloating
 Non anion-gap metabolic acidosis
 Pulmonary aspiration
 Resource distraction from resuscitative and supportive care
 Delayed retrieval to a hospital offering definitive care
Contraindications
 Good outcome assured with supportive care or antidote therapy
 Uncooperative patient
 Inability to place NGT
 Uncontrollable vomiting
 Decreased LOC or expected in subsequent 4 hours
 Ileus or intestinal obstruction
 Intubated and ventilated patient (relative) – hazardous as fluid may pool in oropharynx and flow past
cuff into the lungs
Technique
 Place NGT
 Give AC 1g/kg (upto 50g) via NGT in non-metallic ingestions
 Administer PEG-ELS via NGT at 2L/hr (25ml/kg/hour in children)
 Administer Metoclopramide to minimize vomiting and enhance gastric emptying
 Place patient on commode to accommodate explosive diarrhea
 Continue irrigation until effluent is clear
 Cease WBI if loss of bowel sounds or abdominal distension occurs
 Abdominal x-ray useful to assess effectiveness of decontamination of radio-opaque substances such
as iron and potassium salts
 Expelled packages counted in body-packers.
Enhanced Elimination
Techniques of enhance elimination are employed to increase the rate of removal of an agent with the aim of
reducing the severity and duration of clinical intoxication.
These techniques are useful only in the treatment of poisoning by a few agents that are characterized by:
 Severe toxicity
 Poor outcome despite good supportive care and antidote administration
 Slow endogenous rate of elimination
 Suitable pharmacokinetic properties
Techniques of EE and amenable agents
Multiple-dose activated charcoal
 Carbamazepine
 Dapsone
 Phenobarbitone
 Quinine
 Theophylline
Urinary alkalinisation
 Phenobarbitone
 Salicylates
Hemodialysis and hemofiltration
 Lithium
 Metformin lactic acidosis
 Potassium
 Salicylates
 Theophylline
 Toxic alcohols
 Valproic acid
Charcoal hemoperfusion
 Theophylline
Once decision to initiate a techinique of EE is made, it is important to establish predefined clinical or
laboratory end-points of therapy.
Multiple-Dose Activated Charcoal (MDAC)
Rationale
MDAC fills the entire GIT with AC. This has the potential to enhance drug elimination in two ways:
 Interruption of the entero-hepatic circulation
o For drugs excreted in bile
o And has a relatively small volume of distribution
 Gastrointestinal dialysis
o Drug passes from high concentration intracellularly to low concentration in gut, which is
maintained by adsorption to charcoal
o Only effective if drug is relatively small molecule, lipid-soluble, has small volume of
distribution and low protein binding.
Indications
 Carbamazepine coma
o Most common indication for MDAC
o To reduce duration of ventilation therapy and length of stay in ICU
 Phenobarbitone coma
o Rare
o To reduce duration of ventilation therapy and length of stay in ICU
 Dapsone overdose with methemogobulinemia
o Very rare scenario in Australia
 Quinine overdose
o Though may be used to reduce length of time of toxicity, good supportive care usually
associated with good outcome
 Theophylline overdose
o Hemodialysis more effective and carried out first if available.
Absolute contraindications
 Decreased LOC without prior airway protection
 Bowel obstruction
Complications
 Vomiting (30%)
 Charcoal aspiration
 Constipation
 Charcoal bezoar formation, bowel obstruction, bowel perforation
 Corneal abrasion
 Resource distraction
Technique
 Give initial dose of AC 50g (1g/kg in children ) PO
 Give repeat doses of 25g (0.5g/kg in children) every 2 hrs
 Check bowel sounds before each dose administration, cease if bowel sounds stop
 Review indication and clinical end-points for therapy beyond 6 hours (rarely needed)
Urinary Alkalinisation
Rationale
The production of an alkaline urine pH promotes the ionization of highly acidic drugs and prevents
reabsorption across the renal tubular epithelium thus promoting excretion in the urine. For this method to be
effective the drug must be filtered at the glomerulus, have a small volume of distribution and be a weak acid.
Indications
 Salicylates overdose
o Salicylates are hepatically metabolized and fail to be excreted in acidic urine. In overdose,
metabolism is saturated and elimination half-life is greatly prolonged
o Urinary alkalinisation greatly enhances elimination and is indicated in any symptomatic
patient in effort to reduce duration and severity of symptoms and to avoid progression to
severe poisoning and need for hemodialysis
o Severe established salicylates toxicity is indication for immediate hemodialysis
 Phenobarbitone coma
o May be attempted to reduce length of symptoms
o MDAC first choice of therapy
Contraindications
 Fluid overload
Complications
 Alkalemia
 Hypokalemia
 Hypocalcemia
Technique
 Correct hypokalemia
 1-2mmol/kg sodium bicarbonate IV bolus
 Commence infusion of 100mmol sodium bicarbonate in 1L of 5% dextrose at 250ml/hr
 20 mmol of KCl may be added to maintain normolkalemia
 Monitor serum bicarbonate and potassium q4hrs
 Regularly dipstick urine and aim of urinary pH >7.5
 Continue until clinical and laboratory evidence of toxicity is evidenced
Extracorporeal Techniques of Elimination
A number of these techniques exist and include:
 Hemodialysis
o Intermittent
o Continuous
 Hemoperfusion
 Plasmapharesis
 Exchange transfusion
Indications
 All these techniques are invasive and require specialized staff, equipment and may be associated with
significant complications and are reserved for life-threatening poisonings where a good outcome is
not achievable by other measures
Clinical situations that satisfy above criteria include:
 Toxic alcohol ingestions
o Methanol
o Ethylene glycol
 Theophylline toxicity
 Severe salicylates intoxication
o Chronic intoxication with altered LOC
o Severe established salicylates toxicity
 Severe chronic lithium intoxication
 Phenobarbitone coma
 Metformin-induced lactic acidosis
 Massive valproate overdose
 Massive carbamazepine overdose
 Potassium salt overdose with life-threatening hyperkalemia
Hemodialysis is the most frequently used modality. Intermittent dialysis achieves greater clearance rates than
continuous hemodialysis.
Toxidromes
Type
Etiology
Clinical features
Anitcholinergic
Atropine, Scopolamine,
TCA, antihistamine,
Amantadine,
phenothiazines, Plants
e.g. Datura, nightshade,
Amanita muscaria
mushrooms
 HOT as a HARE – fever
 BLIND as BAT – spasm
of accommodation
(dilated)
 DRY as BONE – dry
MM and skin
 RED as BEET – flushed
skin
 MAD as HATTER –
central anticholinergic
syndrome
 FULL as GOOG –
urinary retention
Cholinergic
Organophosphate
insecticides, carbamate
insecticides, neostigmine,
physostigmine,
mushrooms
Narcotic
Heroin, Morphine,
Methadone
 Salivation and
sweating
 Lacrimation
 Urination
 Diarrhea
 GI pain
 Emesis
 Coma
 Pin point pupils
 Hypoventilation
 CNS depression
Phenothiazines
Chlorpromazine,
prochlorperazine,
haloperidol and others
 Hypotension – antiadrenergic
 Dystonias: torticollis,
oculogyric crisis,
opisthotonus antidopaminergic
 Constricted pupils
Comments
 Will also have
tachycardia and
hypertension
 Treatment usually
supportive
 Physostigmine used in
life-threatening
situations
 Significant tachycardia,
hyperthermia or
seizures resitant to BDZ
indication for PHYS use
 1-2mg IV over 2mins,
onset rapid, duration
30 mins.
 Complications of
bradycardia, heart
block and seizures
 PHYS C/I in TCA OD
 Airway protection
 Ventilation
 Atropine
 Pralidoxime when
indicated
 Most opiates t1/2 36hrs, except
methadone 15-20h and
propoxyphene 12-15h
 Hypothermia and
bradycardia may also
occur
 Acute lung injury and
ARDS
 Naloxone is antidote
but may ppate
withdrawal in abusers
 Anticholinergic,
antiadrenergic and
antidopaminergic
properties
 Benztropine 1-2mg IM
for EPS
 Prolonged QT,
widened QRS
 Dry mouth,
tachycardia anticholinergic
 Tachycardia,
hypotension
 Anticholinergic signs
 CNS depression,
seizures
 QRS widening and RAD
Tricyclic antidepressants
Amitryptline,
Imipramine, Prothiaden,
Doxepin
Sympathomimetics
Adrenergic agonists –
 α – effects –
direct acting – α and β
hypertension,
Dopaminergic agonists
bradycardia, dilated
and indirect acting
reactive pupils,
Noradrenaline transport
palms/feet sweating,
blocking – amphetamines
↓ GI motility, blader
Adrenaline/Norad
contraction
metabolism blocking –
 β – effects –
MAOI and COMT
tachycardia,
inhibitors
vasodilation ±
hypotension, miosis,
↓ GI motility, bladder
relaxation
 euphoria, restlessness
to toxic psychosis and
seizures
Ultrashort – Thiopental,  coma, hypoventilation
Short – Pentobarbital,
 hypotension,
intermediate –
hypothermia
Amobarbital and long
 barbiturate blisters
acting – Phenobarbital
Barbiturates
Hallucinogens
Serotonin syndrome
Psychedelics –
perception altering drugs
– LSD, DMT – act on 5HT2A
Dissociatives – PCP,
ketamine – NMDA
antagonism
Deliraints –
anticholinergics
Cannabinoids – CB-1
receptor agonists
Usually combination
therapy or recent change
in dosing of MAOI, TCA,





α – mimetic effects
delirium
psychosis
hallucinations
seizures
 mental status changes
– elevated mood,
restlessness, insomnia,
 Anticholinergic, antiadrenergic and
quinidine like direct
cardiac effects
 Na-channel blocking
activity responsible for
cardiac toxicity
 supportive treatment
 no specific antidote
available
 seizures and
hyperthermia may
produce
rhabdomyolysis and
myoglobinuria
 depression of neuronal
activity
 enhanced GABAmediated chloride
currents
 MDAC or hemodialysis
for long acting agents
 mainly symptomatic
and supportive therapy
 Sternbach criteria
 Hunter serotonin
toxicity criteria
Neuroleptic Malignant
Syndrome
SSRI, amphetamines,
coma
opioids, rarely SSRI alone  neurologic –
incoordination,
myoclonus, tremor,
shivering, rigidity,
hyperreflexia
 Autonomic
manifestations – fever,
tachycardia, sweating,
tachypnea, diarrhea,
HT or hypotension,
mydriasis
At initiation of
 Clouded mentation
neuroleptic agent,
 Autonomic
antipsychotics
disturbance – fever,
Recent dosage change or
HT, Tachycardia,
medical condition
tachypnea, sweating,
incontinence
 Motor signs - ↑tone,
tremor, dyskinesia,
parkinsonism, dystonia
 Lab findings – CK>500,
WCC>14000
 Supportive care
 Cyproheptadine and
chlorpromazine – HT2
antagonists
 Non-selective HT2
antagonists – atypical
antipsychotics –
olanzapine
 Onset slow compared
to SS
 Treat hyperthermia
aggressively
 Dantrolene,
bromocriptine tried ??
 Volume therapy,
urinary alkalinisation
 Dialysis
Antidotes
Opioid antagonists – Naloxone
This Opioid antagonist is an useful adjunct in the management of Opioid intoxication.
Indications
 Reversal of CNS and respiratory depression caused by Opioid intoxication
 Empiric treatment for coma thought to be secondary to opioids
Contraindications
 Avoid in the opioid-dependent individual unless –
o Significant respiratory depression (RR<8)
o CNS depression (GCS<12)
Mechanism of action
Pure competitive opioid antagonist at µ, κ and δ receptors
Pharmacokinetics
 Poor oral bioavailability with extensive first-pass effect
 Well absorbed after IM,SC or ET administration
 Rapid distribution and onset of action
 Hepatically metabolized with Et1/2 60-90 minutes
 Duration of effect – 20 – 90 minutes
Administration
 Monitored area – monitor RR, GCS and O2 saturation in view of re-sedation
 Dosage variable depending on amount of agonist present. Children can have larger doses without risk
of withdrawal
 Initial bolus of 100µg IV or 400µg IM/SC, larger doses when patient is not opioid dependent
 Repeated doses of 100µg IV q30-60 seconds until spontaneous respiration is re-established
 Doses >400µg rarely needed for heroin OD, partial agonists may require larger doses
 OD with CR and SR morphine tablets or methadone, re-sedation is expected and naloxone infusion
may be necessary
 Commence infusion at 2/3rd of initial dose required/hour. 2mg naloxone in 100ml NS at 5ml/h gives
100µg/h.
 Titrate infusion to response and RR
Therapeutic endpoints
 Normal mental status – in opioid naïve patients
 Maintenance of adequate airway, RR and rousable GCS(>13) – opioid dependent patients
 All patients given naloxone should be monitored for at least 2 hrs after the last dose
Adverse drug effects
 Nil significant in opioid naïve patients
 Dose dependent withdrawal syndrome in opioid dependent patients
 Reassurance, physical and chemical control of disturbed patient may be needed if withdrawal
precipitated
Pitfalls
 Production of acute withdrawal syndrome with large doses
 Inadequate naloxone dose following partial agonist OD
 Failure to recognize the risk for re-sedation with longer acting agents as naloxone effects last only 3090minutes
Oxygen therapy – normobaric and hyperbaric
Oxygen is one of the most widely used therapeutic agents. Oxygen is widely available and commonly
prescribed by medical staff in broad range of conditions to relieve and prevent tissue hypoxia.
Indications for use of HBO
Sufficient clinical evidence is available for the use of HBO in
 carbon monoxide poisoning,
 decompression sickness,
 arterial gas embolism,
 radiation-induced tissue injury,
 clostridial myonecrosis,
 problem wounds,
 crush injury, and
 refractory osteomyelitis
Application
 Normobaric oxygen(NBO) is applied via a variety of masks/ cannulae that allow delivery of FiO2
between 24-90%
 Higher concentration can be delivered via masks with reservoirs, tightly fitting CPAP type masks or
mechanical ventilation
 Hyperbaric oxygen (HBO) at pressures higher than 1 atmosphere absolute can be achieved in single
chamber or large multiplace hyperbaric chamber.
Tissue oxygenation
 Delivery of oxygen to tissues depends on adequate ventilation, gas exchange and circulatory
distribution
 When breathing air at normal atmospheric pressure, most oxygen is bound to hemoglobin and very
little transported dissolved in plasma
 On inspiring increasing FiO2, hemoglobin is completely saturated and dissolved oxygen in plasma
increases but due to oxygen’s poor solubility, NBO at 100% can only provide 1/3 rd of tissue
oxygenation in dissolved state.
 HBO at 3 atm provides enough dissolved oxygen to meet average requirements of resting tissues. This
is part of rationale behind use of hyperoxia and HBO in CO poisoning
Hemodynamic effects
 Temporary increase in blood pressure by increasing PVR secondary to systemic peripheral
vasoconstriction counterbalanced by decrease in heart rate and cardiac output
 Hyperoxia induced vasoconstriction and high blood oxygen tension is considered beneficial in crush
injury and compartment syndrome
 NBO decreases ICP and improves brain oxidative metabolism in patients with severe head injury
 NBO therapy in resuscitation of newborn depressed infants has not shown any benefit with better
results in studies shown with 21% oxygen resuscitation
Effects on inflammation
 Tissue hypoxia activates large number of vascular and inflammatory mediators that trigger local
inflammation and may lead to systemic inflammatory response (SIR) that in many cases culminate in
multiorgan dysfunction and failure
 Hyperoxia appears to exert a simultaneous effect on a number of steps in the proinflammatory
cascades. HBO decreases rolling and adhesion of PMNL in the microcirculation following inflammatory
response of skeletal muscle, small bowel, skin flaps, heart and liver as well as after CO poisoning.
 HBO may also exert effects on the IR by ameliorating tissue hypoxia
Effects on microorganisms and tissue repair mechanisms
 HBO exerts direct bacteriostatic and bactericidal effects mostly on anerobic microorganisms.



HBO therapy restores phagocytosis and augments the oxidative burst that is needed for leukocyte
microbial killing.
The activity of many antibiotics is impaired in hypoxic environments and HBO corrects this.
Direct activity on bacteria (pseudomonas, Escherichia, Clostridium species etc), improvement of
cellular defense mechanisms, synergisitic effects on antibiotic activity, modulation of immune
response and augmentation of mechanisms of tissue repair form the basis for use of HBO as
adjunctive therapy in treating tissue infections and sepsis induced SIR
Toxicity
 The major limiting factor to the liberal use of hyperoxia is its potential toxicity and relatively narrow
margin of safety between effective and toxic doses
 The most obvious toxic manifestations of oxygen are exerted on the RS and CNS
 O2 toxicity is believed to result from the formation of reactive oxygen species (ROS) in excess of the
quantity that can be detoxified by the antioxidant systems in the tissues
 Lungs are the organs exposed to the highest oxygen tensions and thus first organs to respond
adversely
 Pulmonary O2 toxicity is characterized by initial latent period with no symptoms or signs
Acute tracheobronchitis is the earliest clinical syndrome. It does not develop in humans breathing less the
50% FiO2 at normal atmosphere.
 At FiO2 >95% tracheobronchitis develops after a latent period of 4 – 22hrs
 The symptoms may last for a few hours after onset and cessation of HBO therapy and completely
resolves some days after cessation of therapy
 Longer exposures to hyperoxia >48hrs may induce diffuse alveolar damage
 Long exposures to hyperoxia during mechanical ventilation may result in COPD characterized by
marked residual pulmonary fibrosis and emphysema
CNS oxygen toxicity
 Occurs at much higher concentrations than lung injury and usually needs HBO
 CNS toxicity may manifest as generalized tonic-clonic seizures but less severe signs include nausea,
dizziness, sensation of abnormality, headache, disorientation, light-headedness, blurred vision,
tinnitus, twitching
Other
 Cataract formation – rare with lower pressures
 Barotrauma to middle ear, sinuses, teeth or lungs may occur with rapid changes in pressure during
HBO
In conclusion, HBO has specific indications where there are studies which have shown its usefulness, whereas
there is not much evidence out there to support or refute the use of NBO in most clinical situations except
presence of hypoxia.
N-acetylcysteine (NAC)
NAC is the most widely used sulfhydryl donor in the treatment of paracetamol poisoning. Standard therapy
consists of a series of three infusions given over 20 hours. It is completely protective against paracetamolinduced hepatotoxicity when administered within 8hrs of an overdose.
Toxicology Indications
 Acute paracetamol overdose
 Repeated supra-therapeutic paracetamol ingestion
 Paracetamol induced fulminant hepatic failure
 Investigated for use in other poisonings:
o Chemotherapeutic agents
o Paraquat
o Carbon tetrachloride
o Chloroform
o Amanita mushrooms
 Prevention of contrast induced nephropathy
Contraindications
 None
Mechanism of action
NAC prevents N-acetyl-p-benzoquinoneimine (NAPQI)-induced hepatotoxicity by four possible mechanisms
 Increased glutathione availability
 Direct binding to NAPQI
 Provision of inorganic sulfate
 Reduction of NAPQI back to paracetamol
NAC has antioxidant properties which may offer benefit in many other poisonings.
Pharmacokinetics
NAC metabolism is complex. Plasma half life is 6hrs and 30% is eliminated unchanged in urine.
Administration
 Patients carefully monitored during initial dose for anaphylactoid reactions. Cardiac monitoring is not
required after that ime.
o 150mg/kg NAC diluted in 200ml 5%D IV over 15minutes followed by
o 50mg/kg NAC diluted in 500ml of 5%D IV over 16hours followed by
o 100mg/kg NAC in 1000ml of 5%D over 16hrs
 Standard treatment is 20hrs but may be interrupted early if risk of hepatotoxicity is excluded
 Treatment may be continued longer in patients with late presentations, repeated supra-therapeutic
ingestion or ongoing biochemical evidence of hepatotoxicity
Therapeutic endpoint
 Absent or resolving hepatotoxicity as determined by aminotransferases
Adverse drug reactions and their management
 Mild anaphylactoid reactions (15-20%) – mild flushing, rash and angio-edema, treat with
promethazine and restart at slower rate.(controversial if this helps).
Digoxin immune FAB (Digibind)
Presentations
Digoxin-specific immune antigen binding fragments as lyophilized powder 38mg ampoules
Toxicological Indications
 Acute digoxin overdose
o Cardiac arrest
o Life threatening cardiac dysrhythmia
o Ingested dose >10mg (adult) or 4mg (child)
o Serum digoxin level >15nmol/ml (12ng/ml)
o Serum potassium >5mmol/L
 Chronic digoxin poisoning
o Cardiac arrest
o Life threatening or dangerous cardiac dysrhythmia
o Moderate to severe GI symptoms
o Any symptoms in presence of impaired renal function
 Other cardiac glycoside poisoning
o Oleander
o Bufotoxin (cane toad)
Contraindications
 None
Mechanism of action
Digoxin immune FAB binds directly to the free intravascular and interstitial digoxin with much greater affinity
than the Na/K ATPase receptor. A concentration gradient is created for intracellular digoxin to dissociate from
tissue and move to intravascular space where more binding continues.
Pharmacokinetics
 Elimination ½ life of FAB fragments is 12hrs and digoxin bound to FAB fragments is excreted in urine
with elimination ½ life of 16-30hrs
Administration
 Patient in monitored area with full resuscitation capability
 Cardiac monitoring continued until toxicity is reversed
 Calculate dose required, dilute in 100ml NS and administer over 30minsIV
 One ampoule of FAB binds 0.5mg of digoxin
Dosing
 Acute digoxin overdose
o Number of ampoules = ingested dose (mg) X 0.8(bioavailability) X 2
o Unknown dose – 5amps if patient stable and 10amps if unstable with repeat dose in 30 mins
if inadequate response
 Chronic digoxin poisoning
o Number of ampoules = serum digoxin(ng/mL) X body weight(kg) / 100
o Or give two ampoules every 30 minutes until desired response
 Other cardiac glycoside poisoning
o Empiric dosing of 5 amps every 30 mins until reversal of toxicity
o Large dose up to 30 amps have been used with severe yellow oleander poisoning.
Therapeutic end-points
 Restoration of normal cardiac rhythm and conduction
 Resolution of GI symptoms
Adverse drug reactions and their management




Note


Hypokalemia
Allergy
Exacerbation of underlying cardiac failure
Loss of rate control in c/o AF
After administration of digibind, digoxin levels may appear very high, because assays measure both
free and FAB-bound digoxin, so do not target serum levels
Hyperkalemia due to digoxin toxicity should be primarily treated with digibind and not calcium, since
digoxin causes increased cytosolic calcium levels and can cause cardiac standstill
Paracetamol Poisoning – acute overdose
Risk assessment
 Life threatening hepatotoxicity is uncommon and fatalities from paracetamol poisoning are rare
 Threshold for hepatic injury in adults is variable but considered to be >150mg/kg (>10g)
 Risk of hepatic injury is predicted by plotting a serum paracetamol level taken 4-15 hrs later on the
Precott or Rumack-Matthew nomogram
 The probability of hepatotoxicity (defined as AST or ALT >1000IU/L) is
o 1-2% if 4hr level <1320µmol/L
o 30% if 4hr level is 1320-1980µmol/L
o 90% if 4hr level is >1980µmol/L
 The risk of hepatic injury with NAC is determined primarily by time from overdose to commencement
of NAC
o Survival 100% if NAC commenced <8hrs since ingestion
o Benefit reduced if NAC commenced 8-24hrs following ingestion
o Benefit not established after 24h of ingestion
o In fulminant hepatic failure NAC administration reduces cerebral edema, inotrope
requirements and mortality
 Risk assessment difficult if time of ingestion not known
 Patients who present >8hrs post ingestion with deranged LFTs assumed to have early paracetamol
induced hepatotoxicity
 Patient presenting 24hrs after ingestion with no LFT derangement and no detectable paracetamol in
serum has little risk of developing hepatotoxicity
 There are no reports of death following accidental paracetmol exposures in children under 8yrs.
Ingestion of <200mg/kg does not warrant decontamination, referral to hospital, serum paracetamol
level, LFTs, antidote treatment or follow up.
Toxic mechanism
 In therapeutic doses most APAP is metabolized in liver by glucuronidation or sulfation to result in nontoxic metabolites, very little APAP undergoes CYP-mediated N-hydroxylation to form N-acetyl-pbenzoquinoneimine(NAPQI), a highly reactive metabolite. NAPQI normally reacts with sulphydryl
groups of glutathione(GSH), which renders it harmless
 In large or toxic doses, NAPQI id formed in sufficient amounts to deplte hepatic GSH and thus
contributes to toxicity by hepatic membrane peroxidation and sulfation of cellular proteins
 The hallmark of APAP-induced hepatic injury is centrilobular necrosis.
Toxicokinetics
 APAP is well absorbed orally, peak levels within 45mins; Vd of 0.9L/kg
 80% undergoes hepatic glucuronidation and sulfation which is then excreted in urine
 5 – 15% undergoes oxidation by CYP450sytem to form NAPQI
 NAPQI usually binds intracellular glutathione and cysteine forming mercaptouric metabolites excreted
in urine
Clinical features
In patients who develop hepatic injury, four clinical phases are described
Time
Clinical features
Phase 1 (<24hrs)





Phase 2 (1 – 3 days)
Frequently asymptomatic
Nausea and vomiting
RUQ tenderness
ALT and AST ↑ rapidly
In survivors ALT, AST rapidly return to baseline
Phase 3 (3 – 4 days)
Phase 4 (4 days – 2 weeks)
 PT and INR abnormal as AST, ALT peak
 ↑Bilirubin and impaired renal function may also
occur
 Severe cases hepatotoxicity →fulminant hepatic
failure → coagulopathy, jaundice, encephalopathy
and MODS
 Lactic acidosis, renal failure despite resuscitation
 Recovery phase in survivors during which hepatic
structure returns to normal
Investigations
Bedside
 BSL
 ECG – for co-ingestions
 ABG – if indicated due to co-ingestions and delayed presentations with severe MODS
Specific investigations
 Serum paracetamol levels
o At 4hrs or more, if time of ingestion is known to establish risk of hepatotoxicity and need for
treatment
o If NAC commenced within 8hrs of ingestion, single timed level of >4hrs is usually enough to
direct therapy
o If NAC commencement delayed by >8hrs, then serial AST, ALT levels + PT/INR may be needed
to monitor evidence for hepatic injury
 Coagulation studies, platelet counts, renal function and acid-base status indicated if hepatotoxicity
established
Management
Resuscitation, supportive care and monitoring
 ABC approach only required if delayed presentation with sever hepatotoxicity or in c/o significant coingestions
 Correction of hypoglycemia should take priority in these patients
 General supportive care and monitoring measures as indicated
 Patients with rising AST, ALT should have close monitoring of vitals, BSL and fluid balance
Decontamination
 Oral AC is not life-saving
 May be offered to cooperative adults who present within 1hr of ingestion, which case it may prevent
further absorption and reduce APAP levels at 4hrs and avoid need for NAC administration
 Not indicated for small children since NAC is available and accidental overdoses is rarely lethal
Antidotes
 IV NAC indicated in all patients whose risk assessment suggests potential for poor outcome and in
late-presenters with clinical and biochemical evidence of hepatic injury
 Presentation <8hrs following a defined time of ingestion and if serum APA level available within 8hr
window, NAC administration deferred until results available
 Presentation 8-24 hrs following ingestion – NAC commenced immediately and ceased once APAP level
available and plotted on nomogram
 Unknown time of ingestion – serum APAP detectable or suspected – commence NAC immediately and
cease after details available or full 20hrs course

Presentation >24hrs post-ingestion – NAC only indicated if serum APAP still detectable and if AST, ALT
elevated and continued until levels falling.
Disposition and follow up
 Patients in whom NAC commenced within 8hrs may be considered for medical discharge at the end of
20hrs infusion
 Patients in whom NAC is commenced after 8hrs of ingestion, NAC is continued until ALT, AST levels
are falling
 In c/o established hepatic failure with rising INR and ALT, AST; transfer to a liver transplant service
should be considered.
 Transfer to a liver transplant service should be considered if:
o INR >3.0 at 48hrs or INR >4.5 at any time
o Oliguria with creatinine level >200µmol/L
o Acidosis pH<7.3 after resuscitation
o Systolic hypotension <80mmhg
o Hypoglycemia
o Severe thrombocytopenia
o Encephalopathy
Specific situations
 In case of staggered OD within few hours, consider first dose time as time of ingestion and calculate
nomogram from there. Though very conservative, this approach is less likely to cause increased
morbidity
 There is no data to show a lower threshold level is needed for patients with hepatic disease or at
increased risk of hepatotoxicity
 In c/o extended release preparations, if total dose is >150mg/kg commence NAC and conduct 4hr and
8 hr levels and cease therapy if both levels below nomogram threshold.
Paracetamol – Repeated supratherapeutic ingestion(RSI)
Risk assessment
 Available nomograms are not useful
 Risk assessment based on dose history and biochemical testing
 Biochemical testing indicated for adults and children >6yrs if:
o 10g or >200mg/kg ingested over 24-hr period
o 6g or 150mg/kg/24h for 48hrs or longer
 Patients with reduced hepatic function with >4g or >100mg/kg/24h
 Risk assessment based on APAP and ALT, AST levels
o ALT or AST <50 IU/L and APAP level <66µmol/L – good prognosis
o ALT or AST >50IU/L and APAP level>66µmol/L – higher risk group
 Children – consider biochemical risk assessment if;
o >200mg/kg/24h for 1 day or more
o >150mg/kg/24 for preceding 48hrs
o >100mg/kg/24h for preceding 72hrs
Toxic mechanism
Repeated supratherapeutic ingestions may lead to depletion of cellular stores of glutathione and thus increase
risk of APAP induced hepatic injury.
Clinical features
In patients developing significant hepatotoxicity, clinical features may occur in similar phases as with acute
overdose
Investigations
 Serum APAP, AST, ALT levels – initially to assess risk of hepatotoxicity
 LFTs, BUN, PT/INR, ABG and BSL – to monitor clinical course for patients with confirmed hepatic injury
Management
Resuscitation, supportive care and monitoring
 Only required if presentation very delayed with established hepatic failure. In such case priorities are
o Airway, breathing, circulation
o Correction of hypoglycemia
o Correction of coagulopathy if indicated by bleeding
 In patients with INR>2.5 close vitals, BSL and fluid status monitoring is justified
Decontamination
 Not indicated for repeated supraterapeutic ingestion scenario
Enhanced elimination
 Not useful
Antidotes
 NAC indicated immediately if there are clinical features of hepatitis, h/o RSI paracetamol otherwise
commenced after biochemical risk assessment conducted
 IV NAC once instituted continued for atleast 8hrs and biochemical markers rechecked – if improved,
NAC ceased, if deterioration – continue NAC 16hrs and repeat until therapeutic end-points reached
Disposition and follow up
 Patients with evidence of hepatotoxicity – admitted for NAC infusion, LFT monitoring. May be
discharged once well and transaminases improving.
 Patients with signs of fulminant hepatic failure require ICU admission and consideration for referral to
liver transplant unit
Salicylate poisoning
Salicylate poisoning can be classified into acute and chronic intoxication. Morbidity and mortality are greater
with chronic intoxication.
Risk assessment
 Severity of clinical features following aspirin overdose is dose related and progressive over hours
 5g methyl salicylates = 7.5g acetylsalicylate
Dose related risk assessment with acetylsalicylic acid
Dose
Effect
>150mg/kg
Minimal symptoms
150 – 300mg/kg
Mild to moderate intoxication, salicylism with
hyperpnoea, tinnitus, vomiting
>300mg/kg
Severe intoxication, metabolic acidosis, altered
mental state, seizures
>500mg/kg
Potentially lethal

Children rarely ingest a dose of aspirin sufficient enough to cause toxicity, but smaller dose may cause
worse symptoms in children.
Toxic mechanism
 Irreversible inhibition of cyclo-oxygenase enzymes → decreased prostaglandin synthesis
 Stimulation of respiratory centre → hyperventilation and respiratory alkalosis
 Uncoupling of oxidative phosphorylation → accumulation of lactic acidosis → metabolic acidosis
 Promotion of fatty acid metabolism →generation of ketone bodies → contribution to metabolic
acidosis
Toxicokinetics
 Rapidly absorbed following oral administration, highly protein bound with Vd 0.1-0.3L/kg
 Absorption may be delayed with enteric coated forms or when large tablet masses form within the
stomach – bezoars
 In overdose, protein binding is saturated with increased free salicylate levels → acidemia → more
unionized salicylate movement into extravascular spaces including CNS
 Elimination ½ life usually 2-4hrs, may increase up to 24hrs in overdose
 First-order kinetics of metabolism change to zero-order kinetics as metabolic pathways become
saturated in toxic doses
 High urinary pH promotes urinary salicylate excretion as greater proportion of salicylate is ionized and
cannot be reabsorbed
Clinical features
Acute salicylate intoxication
 Onset of symptoms progressive over hours and may not be evident until 6-12hrs after ingestion,
deterioration can then be very rapid
 Gastrointestinal
o Nausea, vomiting
 CNS
o Tinnitus, decreased hearing, vertigo
o CNS stimulation, agitation, seizures
o Cerebral edema and death
 Acid-base disturbance
o Respiratory alkalosis
o Elevated anion gap metabolic acidosis
o Worsening acidemia due to lactic acidosis late and s/o severe insult
 Other
o Hyperthermia, hyper/hypo- glycemia, Hypokalemia
Chronic salicylate intoxication
 Difficult to diagnose clinically
 Most commonly elderly patients
 Non-specific symptoms such as confusion, delirium, dehydration, fever and unexplained metabolic
acidosis
 Cerebral and pulmonary edema more common than acute poisoning
Investigations
Screening tests in deliberate self-poisoning
 12-lead ECG, BSL and paracetamol level
Specific investigations
 Salicylate levels
o Therapeutic level 1.1 – 2.2mmol/L
o Poor correlation between levels and severity of toxicity
o Serial levels helpful to diagnose ongoing absorption, bezoar formation or SR tablets
 Arterial blood gases
o Detect and monitor acid-base disturbances
Management
Resuscitation, supportive care and monitoring
 General supportive care measures as dictated by the level of consciousness, if patient is intubated,
then ensure hyperventilation to maintain respiratory alkalosis
 Control seizures with IV benzodiazepines
 Close fluid balance monitoring and replacement to avoid dehydration or overload
Decontamination
 PO AC 50g for up to 8hrs post ingestion if acute overdose>150mg/kg
 If patient intubated give 50g AC NG
 If salicylate levels continue to rise repeat 50g AC in 4hrs
Enhanced elimination
 Urinary alkalinisation is indicated in symptomatic salicylate poisoning
 Hemodialysis is indicated for removal of salicylate only if
o Urinary alkalinisation not feasible
o Serum salicylate level rising >4.4mmol/L despite all measures
o Severe toxicity with altered mental status, acidosis and renal failure
o Very high serum salicylate levels
 Acute - >7.2mmol/L
 Chronic - >4.4mmol/L
o Elderly patients with lower thresholds
Antidotes
 None available
Disposition and follow up
 All symptomatic patients require admission for careful monitoring, enhance elimination and until end
of therapy ( levels 1.1-2.2mmol/L) and when acid-base abnormalities have resolved
 Patients with significant toxicity require ICU or HDU admission
 Refer patients early for dialysis if ingestion is >300mg/kg
Opioids
Buprenorphine, Codeine, Dextropropoxyphene, Fentanyl, Heroin, Hydromorphone, Methadone, Morphine,
Oxycodone, Pethidine and Tramadol.
Risk assessment
 Life-threatening CNS and respiratory depression can occur just above the analgesic dose
 Opioid use in naïve patients and co-ingestion of other CNS depressants e.g. ethanol, BDZ,
antidepressants increases the severity of CNS depression and likelihood of fatal outcome
 Children: opioid intoxication is leading cause of poisoning death in children. Ingestion of single adult
dose of opioid by child can cause respiratory arrest. Ingestion of >5mg/kg of codeine can cause
respiratory arrest and a single tablet of Tramadol can cause seizures in children.
Toxic mechanism
 Agoinst activity at µ-receptors is responsible for euphoria, analgesia, physical dependence, sedation
and respiratory depression.
 Actions at other receptors are responsible for side effects –
o Dopamine receptors – nausea and vomiting
o Peripheral µ-receptors – constipation
o Histamine release – pruritus
o Seizures
Toxicokinetics
 Oral absorption of opioids is variable. Most (except SR, CR) are absorbed rapidly.
 Vd are usually large for most opioids. Most undergo hepatic metabolism to form metabolites which
are excreted in urine.
 Most metabolites retain some opioid activity
Clinical features
 Classic opioid toxidrome consists of:
o Central nervous system depression
o Respiratory depression (rate and depth)
o Miosis
 Duration of effects depend on agent used – heroin 6hrs, methadone 24hrs.
 Death is caused by loss of protective airway reflexes and apnea
 Nausea and vomiting occur – promoting pulmonary aspiration
 Tachycardia – due to hypercapnia and hypoxia
 Hypothermia, skin necrosis, compartment syndrome and rhabdomyolysis may complicate non-lethal
intoxication with prolonged immobilization
 Dextropropoxyphene intoxication is also associated with seizures, hypotension and ventricular
dysrhythmias
 Tramadol overdose rarely causes respiratory depression, but frequently causes tachycardia, agitation
and seizures
 Tramadol and pethidine are associated with serotonin syndrome
Investigations
Screening test in deliberate self-poisoning
 ECG, BSL and paracetamol level
Specific investigations
 No specific drug levels are available readily or useful in management
 Specific other investigations such as CXR, ABG etc are indicated to diagnose and monitor
complications
Management
Resuscitation, supportive care and monitoring
 Attention to airway, breathing and circulation are paramount for successful outcome in management
of these patients
 Close clinical and physiological monitoring is indicated for all patients with opioid toxidrome
 Tramadol seizures are managmed with BDZ
 In rare event of arrhythmias associated with dextropropoxyphene intoxication – resuscitation includes
serum alkalinisation with sodium bicarbonate
Decontamination
 Since opioids are likely to cause respiratory depression with high risk for vomiting, AC is not indicated
for use in opioid intoxication.
 Good outcome is expected through good supportive care and antidote administration
 Only indicated in patient presenting early with overdose from controlled release preparations
Enhanced elimination
 Not useful
Antidotes
 Respiratory and CNS depression can be reversed with titrated doses of naloxone
Disposition and follow up
 Period of observation and monitoring is required to detect CNS depression. 4hrs usually adequate
with most oral preparations
 In case of CR,SR preparations, at least 12hrs of monitoring is indicated
 Tramadol intoxication – observe 12hrs for seizure risk
 Ward or HDU admission depending on the level of CNS/ respiratory depression
 Children admitted for at least 12hrs in case of any opioid ingestion and never discharged at night
 Patients requiring naloxone infusion require HDU admission and those requiring intubation require
ICU.
Anticholinergic poisoning
Etiology
Anticholinergic agents
Antiparkinsonian drugs
Benztropine, amantadine
Antihistamines
Promethazine, diphenhydramine
Antitussives
Dextromethorphan
Antidepressants
TCA
Antipsychotic agents
Butryphenones and phenothiazines
Haloperidol, chlorpromazine
Atypical antipsychotics
Olanzapine, quetiapine
Anticonvulsants
Carbamazepine
Motion sickness agents
Hyoscine
Anti-muscuranic agents
Atropine, glycopyrrolate
Plants
Selected mushrooms, Datura species
Clinical features
Anticholinergic syndrome is best characterized as an agitated delirium associated with variable signs of
peripheral muscarinic blockade.
Central
Peripheral
Agitated delirium characterized by:
 Fluctuating mental status
 Confusion
 Restlessness
 Fidgeting
 Visual hallucinations
 Picking at objects in air
 Mumbling slurred speech
 Disruptive behavior
Tremor
Myoclonus
Coma
Seizures








Mydriasis
Tachycardia
Dry mouth
Dry skin
Flushing
Hyperthermia
Sparse or absent bowel sounds
Urinary retention
Risk of complications from anticholinergic syndrome include:
 Injury to self and others
 Dehydration
 Hyperthermia
 Rhabdomyolysis
 Pre-renal failure
 Pulmonary aspiration and
 Multi-organ failure
Differential diagnosis of anticholinergic syndrome
 Neurotrauma
 Encephalitis
 SAH
 Epilepsy
 Sepsis
 Wernicke’s encephalopathy
 Hypoglycemia, hyponatremia
 Serotonin syndrome and NMS
Management
Resuscitation
1. Attention to airway, breathing and circulation
2. Treat seizures with BDZ
3. Detect and treat hypoglycemia/ hyperthermia
Risk assessment
 Development of anticholinergic syndrome is anticipated with potent agents
 Symptoms usually manifest within the first few hours since ingestion
 Duration of delirium is extremely variable and may persist for days in some circumstances e.g.
benztropine or carbamazepine
Supportive care
 Manage in quiet but well lit area with minimal stimulation
 Reassurance
 IV fluid resuscitation to maintain urine output >0.5ml/kg/hr
 IDC if urinary retention
 Treat agitation with diazepam 5-10mg PO or IV q15mins until quiet but rousable
 One-on-one nursing usually required to maintain levels of supervision, reassurance and sedation
 Avoid using other drugs with know anticholinergic effects e.g. Haloperidol
Investigations
Screening and specific tests
 ECG, serum paracetamol levels if self-poisoning suspected also in case of OTC preparation used which
may contain paracetamol
 Specific drug levels may be indicated e.g. carbamazepine
 EUC
 CK
 Consider further testing to diagnose or monitor complications e.g. CXR, ABG
Decontamination
 Dictated depending on agent used e.g. MDAC may be needed for carbamazepine
Enhanced elimination
 Depends on agent ingested but mostly not useful
Antidotes
 Physostigmine- centrally acting acetylcholinesterase inhibitor – used in selected cases of
anticholinergic delirium
 w/f cardiac complications associated with its use.
 It may also be used to confirm diagnosis to exclude other causes
Cholinergic Syndrome
The cholinergic syndrome is the result of increased acetylcholine activity at central and peripheral muscarinic
and nicotinic receptors and is potentially lethal.
Cholinergic syndrome arises due to either
 Acetylcholinesterase enzyme inhibition or
 Direct agonist action at the receptors
Most cases of cholinergic syndrome are due to poisoning from organophosphate or carbamate pesticides.
Acetylcholinesterase inhibition
Acetylcholine agonists
Organophosphate compounds(OPC)
 Chlorpyrifos
 Fenthion
 Dimethoate
 Malathion
Carbamate insecticides
 Aldicarb
Chemical warfare agents
 Tabun(GA)
 Sarin (GB)
 Soman (GD)
Alzheimer’s dementia agents
 Donepezil
 Tacrine
Myasthenia gravis agents
 Neostigmine
 Edrophonium
 Physostigmine
Muscuranic agonists
 Acetylcholine
 Betanechol
 Pilocarpine
Nicotine
Mushrooms
Clinical features
Excessive stimulation of cholinoreceptors produces a constellation of CNS, autonomic and neuromuscular
effects
Clinical features of cholinergic syndrome
CNS
Neuromuscular
Parasympathetic
muscarinic
Sympathetic muscarinic
Agitation
Lethargy
Confusion
Respiratory depression
Coma
Seizure
Fasciculation
Muscle weakness
Miosis
Lacrimation
Salivation
Bronchorrhoea
Bronchoconstriction
Abdominal cramping
Vomiting
Diarrhea
Urinary incontinence
Bradycardia
Mydriasis
Sweating
Tachycardia
Hypertension
Symptoms of complications may occur which include:
 Rapid onset of respiratory failure
 Seizures
 Dehydration
 Medium and long term neurological sequelae of OPC intoxication
Management
Resuscitation
 Attempts at decontamination should not delay resuscitation efforts
 Attention to airway, breathing and circulation is paramount. Early control of pulmonary secretions
and administration of oxygen is key to survival
 Give oxygen
 Administer atropine if objective signs of muscarinic exces such as cough, dyspnea are present.
Repeated escalating doses of atropine are given until drying of respiratory secretions is achieved
 Control seizures with BDZ
 If adequate oxygenation is not achieved with O2 and atropine, make decision to intubate early.
Risk assessment
 Depends on underlying cause
 Deliberate self-poisoning with OPC usually is life threatening and need for large doses of atropine
should be anticipated
Supportive care
 In insecticide poisoning, staff should manage patient in well ventilated room using universal
precautions. Special PPE is not required.
 Reassurance
 Commence early fluid resuscitation, since early dehydration very likely
 Early IDC insertion and close fluid balance monitoring
Investigations
Screening tests
 ECG, serum paracetamol level if indicated for self-poisoning
 Specific testing dictated by individuala gent – cholinesterase levels
 CXR
 ABG
 Electrolytes and renal function
 Consider further testing to address other possible diagnoses, if in doubt.
Decontamination
 Decontamination efforts should not delay resuscitation efforts
Enhanced elimination
 Determined by agent and risk assessment
Antidotes
 Atropine – large doses may be needed and should be given until oulmonary secretions are dry
 Pralidoxime –
o Indicated for organophosphate poisoning, carbamate and nerve agent poisoning
(controversial topic)
o Pralidoxime reactivates acetylcholinesterase that has been inhibited by binding to OPC. It is
only effective before aging of the acetylcholinesterase-OP complex has occurred, after which
the enzyme complex cannot be reactivated.
o Atropine only relieves the muscarinic symptoms, whereas PAM can improve both. Thus a
improvement in muscle strength may be noted after PAM administration (10-40mins)
o Initial dose of 2g in 100ml of NS IV over 15 mins, followed by 500mg/hr infusion. Infusion
ceased after 24hrs if symptoms improve, with close monitoring for recurrent symptoms. If
recurrence occurs or symptoms continue PAM infusion restarted
o If available, red cell cholinesterase activity assays should be conducted and if activity
maintained 4-6hrs after cessation of infusion, then treatment complete.
o Many days of therapy may be required for severe poisoning
Alcohols
Ethanol
Ethanol causes CNS depression that is synergistic with other CNS depressants and potentially lethal.
Risk assessment
 ETOH causes rapid, dose-related CNS depression with high degree of inter-individual variability
 Dose may be calculated as number of standard drinks consumed – each standard drink has 10g of
ETOH which is
o 375ml can of mid-strength beer (3.5%)
o 100ml glass of wine
o 30ml shot of spirit
 Co-ingestion of other CNS depressants increases risk of respiratory depression
 Seizures may occur in the setting of ethanol intoxication and withdrawal
Toxic mechanism
 Augmentation of GABA-A receptor complex
 ETOH causes dose-dependent cardiovascular depression and impairs gluconeogenesis, thus causing
hypoglycemia and hypotension.
Toxicokinetics
 Rapidly absorbed following oral administration, distributes readily across body water – 0.6L/kg
 Oxidized by cytosolic and microsomal CYP450 alcohol dehydrogenases to form acetaldehyde which is
then metabolized to acetyl coA
 Above serum concentration of 4mmol/L, zero-order kinetics apply. Serum ethanol levels decrease by
approximately 4mmol/hr or 0.02%/hr.
Clinical features
 Progressive with increasing degrees of intoxication
 Disinhibition, emotional lability and euphoria
 Nystagmus, ataxia and slurred speech
 Agitation, aggression and disorientation
 Nausea and vomiting
 Tachycardia, hypotension and hypothermia
 Coma with loss of airway protective reflexes, respiratory depression and hypotension
 Improvement seen in 2-4hrs but may take 6-12hrs before ambulation
Investigations
Screening tests in deliberate self-poisoning
 ECG, BSL and paracetamol level
Specific investigations
 Serum, blood and breath alcohol levels
o Elevated ethanol levels do not necessarily correlate with clinical picture due to wide interindividual variability
 ABG – if significant hemodynamic or deranged LOC
Management
Resuscitation, supportive care and monitoring
 Attention to airway, breathing and circulation are paramount
 Basic resuscitative measures ensure survival of most patients
 Give thiamine 100mg PO/IV in patients with h/o chronic alcohol use
 Clinical and physiological monitoring as indicated
 Monitor for urinary retention and IDC if required
Decontamination
 AC does not work for alcohol ingestions
Enhanced elimination
 Hemodialysis can rapidly remove ethanol from circulation but is rarely indicated as good supportive
care is usually enough
Antidotes
 None available
Disposition and follow up
 Patients with mild CNS depression managed supportively in ward environment until cooperative,
clinically well, eating well, passing urine and oriented.
 Patients with significant CNS depression may need intubation and admission to ICU
 Where appropriate, patients counseled regarding ethanol abuse prior to discharge and options for
quitting given.
Alcohol: Ethylene glycol
Ethylene glycol is a toxic alcohol and deliberate self-poisoning is usually lethal without timely intervention
Sources
 Radiator coolants and antifreeze – 20-98%
 De-icing solutions
 Solvents
 Brake fluids
Risk assessment
 Ingestion of >1ml/kg is potentially lethal
 Deliberate self-poisoning are considered lethal
 Co-ingestion of ethanol complicates risk assessment
 Dermal and inhalation exposure does not lead to intoxication
Toxic mechanism
 Ethylene glycol has similar CNS effects as ethanol
 The toxic effects are due to its metabolites – glycolic acid and lactate – metabolic acidosis
 Calcium oxalate crystals form in tissues including renal tubules, myocardium, muscles and brain →
hypocalcemia
 Acute oliguric renal failure occurs due to nephrotoxic effects of glycolic acid and oxalate crystals
Toxicokinetics
 Rapidly absorbed after oral ingestion, peak concentration within 1-4hrs
 Distributed across body water and rapid CNS penetration
 Ethylene glycol metabolized by alcohol dehydrogenase and aldehyde dehydrogenase to
glycoaldehyde and glycolic acid → metabolized to glyoxylic acid and oxalic acids
 Elimination ½ life 3hrs. ethanol at 11-22mmol/dl competitively inhibits ADH preventing metabolism of
EG and elimination ½ life increases to 17hrs
 EG is exclusively excreted by kidneys
Clinical features
 Initial clinical features develop within first 1-2hrs – similar to ethanol intoxication → euphoria,
nystagmus, drowsiness, nausea and vomiting
 Progressively severe features develop over subsequent 4-12hrs
o Dyspnea, tachypnea, tachycardia, HT and decreased LOC progressing to
o Shock, coma, seizures and death
 Flank pain and Oliguria → acute renal failure
 Late cranial neuropathies described 5 – 20 days later
Investigations
Screening test in c/o deliberate self-poisoning
 ECG, BSL and paracetamol level
Specific investigations
 EUC, serum lactate, serum osmolality and ABG
o Anion-gap acidosis, ↑lactate, ↑osmolar gap
o Venous bicarbonate may be used a surrogate marker of intoxication
o Anion-gap acidosis with elevated lactate ± elevted osmolar gap and rising creatinine is
pathognomic of EG intoxication
 Breath or serum ethano level
o Required to determine if there is co-ingestion and also to titrate ethanol treatment
 Serum ethylene glycol level
o Rarely available in clinically relevant timeframe
 Urine microscopy
o Calcium oxalate crystals – pathognomic of EG intoxication, but absence does not rule out
diagnosis
Management
Resuscitation, supportive care and monitoring
 Attention to ABC of paramount importance
 Patients with severe intoxication are acidemic with some respiratory compensation
o Intubate and hyperventilate to maintain some respiratory alkalosis
o Consider IV sodium bicarbonate 1-2mmol/Kg to prevent worsening acidemia whilst awaiting
Hemodialysis
 Treat seizures with BDZ
 Detect and correct hypoglycemia, hyperkalemia and hypomagnesemia
 Correct hypoclacemia only if refractory seizures or prolonged QT
 Monitor fluid balance and urine output
Decontamination
 Not useful and not indicated
Enhanced elimination
 Hemodialysis is the definitive management of EG intoxication with elimination rates of EG reduced to
2.5-3.5hrs
 Lactate free dialysates with added bicarbonate may assist correction of acidemia
 Indications for Hemodialysis
o h/o large EG ingestion and osmolar gap >10
o acidemia with pH<7.25
o EG level >8mmol/L
 End-pints for Hemodialysis
o Correction of acidosis
o Osmolar gap <10
o EG level ,3.2mmol/L
 ABG and EUC repeated q4hrs for 12hrs following cessation of Hemodialysis
Antidotes
 Ethanol and fomepizole – used as a temporizing measure until Hemodialysis instituted
 Fomepizole not available in Australia
Disposition and follow up
 Children who are clinically well with venous bicarbonate >20meq/L at 4hrs may be discharged
 Similar level also considered for adults with accidental ingestion
 All symptomatic patients and those with deliberate ingestion assumed to have lethal ingestion and
admitted for further evaluation


If renal failure established, dialysis may have to continue for weeks until return of renal function
Survivors of severe intoxications followed up to exclude development of cranial neuropathies
Alcohol: Methanol
Sources
 Chemical applications in industry and science
 Solvent in thinners, varnishes, paints and enamels
 Model aeroplane fuel
 Fuel additive
 Dyes and stains
 Wood alcohol and spirits
Risk assessment
 Ingestion of >0.5ml/kg is potentially lethal
 Deliberate self-poisonings are assumed to be potentially lethal
 Co-ingestion of ethanol complicates risk assessment
 Dermal and inhalation exposure does not lead to methanol intoxication
 Children: >0.25ml/kg ingestion causes toxicity i.e. 2.5ml in 10kg toddler
Toxic mechanism
 Production and accumulation of formic acid produces severe anion-gap acidosis and direct cellular
toxicity due to inhibition of cytochrome oxidase
 Retinal injury and edema leads to blindness
 Subcortical white matter hemorrhages and putamenal edema classically occur in brain
 Lactic acidosis occurs late due to inhibition of oxidative metabolism
Toxicokinetics
 Rapidly absorbed with peak levels within 30-60minutes
 Metabolized by liver ADH to formaldehyde which is further metabolized by aldehyde dehydrogenase
to formic acid
 Elimination ½ life is 24hrs. ethanol in serum concentration of 22mmol/L competitively inhibits ADH
and thus increases methanol elimination ½ life to 43hrs
Clinical features
 Mild CNS depression as with ETOH occur within an hour of ingestion
 Following a latent period of 12-24hrs, symptoms of headache, vertigo, dyspnea, blurred vision and
photophobia develop
 Features of severe intoxication include tachypnea, drowsiness and blindness
 Progressive obtundation leading to coma and seizures due to cerebral edema
 Recovery from CNS toxicity frequently causes extrpyramidal movement disorders
Investigations
Screening tests
 ECG, BSL and paracetamol leve
Specific investigations
 EUC, serum lactate, serum osmolality and ABG
o Anion gap acidosis, lactic acidosis and elevated osmolar gap
o Venous bicarbonate a surrogate marker of intoxication if ABGs not available
 Serum/ breath ethanol level – to detect co-ingestion
 Serum methanol levels – not readily available to be useful clinically
Management
Resuscitation, supportive care and monitoring
 Attention to airway, breathing and circulation paramount


If intubation needed maintain hyperventilation with respiratory alkalosis to avoid further acidosis
If worsening acidosis consider 1-2mmol/kg sodium bicarbonate and infusion until institution of
dialysis
 Systemic acidosis enhances formic acid inhibition of cytochrome oxidase. Raise pH>7.3 with
bicarbonate aliquots of 50mmol
 Treat seizures with BDZ
 Detect and correct hypoglycemia
 Closely monitor fluid balance and urine output
 Co-factor therapy – folinic acid 2mg/kg IV q6hrs until definitive treatment instituted
Decontamination
 Not helpful/indicated
Enhanced elimination
 Hemodialysis – definitive treatment for methanol intoxication – removes methanol and formic acid
correcting acidosis
 Lactate-free dialysate with bicarbonate added may assist correction of acidemia
 Indications for Hemodialysis
o Any patient needing ADH blockade
o Acidosis <7.3
o Visual symptoms
o Renal failure
o Deterioration in vitals or electrolyte status
o Methanol level >16mmol/L
 End-points for Hemodialysis
o Correction of acidosis
o Osmolar gap<10
o Methanol level <6mmol/L
 ABG and EUC repeated q4hrs for 12hrs following cessation of Hemodialysis to confirm reversal
Antidotes
 Ethanol and fomepizole can be used prior to initiation of definitive therapy…
Disposition and follow up
 Children if clinically well and serum bicarbonate >20meq/L at 8hrs may be discharged
 Accidental ingestions in adults with venous bicarbonate >20meq/L at 8hrs and serum ethanol levels
undetectable may be discharged
 All symptomatic patients considered to have lethal intoxication and admitted to hospital and
transferred to appropriate service depending on level of care needed – usually HDU/ICU.
 If dialysis is not available, plan for early transfer when a provisional diagnosis of methanol poisoning is
made
Anticonvulsants
Carbamazepine
Carbamazepine is an anticonvulsant. Deliberate self-poisoning results in predictable dose-dependent CNS and
anti-cholinergic effects. Management is supportive with selected use of enhance elimination techniques.
Risk assessment
 Clinical features are dose-dependent and onset varies depending on type of preparation – immediate
vs. controlled release
 In large overdoses – anticholinergic signs may predominate prior to coma
 Following massive ingestions, coma is anticipated to last several days due to on-going absorption,
slow-elimination and active metabolite
Dose
Effect
20-50mg/kg
Mild-moderate CNS and anticholinergic effects
>50mg/kg
Fluctuating mental status with intermittent agitation
and risk of progression to coma within 12hrs
Risk of hypotension and cardiotoxicity with large
doses
 Carbamazepine is tertogenic and overdose in early pregnancy requires antenatal assessments
 Single tablet of 400mg is capable of causing significant intoxication in toddlers
Toxic mechanism
 Structurally similar to imipramine
 Inhibits inactivated sodium channels thus preventing further action potentials
 Nor-adrenaline uptake inhibitor, antagonist at muscarinic, nicotinic, NMDA and adenosine receptors
Toxicokinetics
 Slowly and erratically absorbed
 Large overdoses → ileus due to anticholinergic effects → on-going erratic absorption for days
 Small Vd 0.8-1.2L/kg undergoing hepatic mtaboilsm by CYP450 3A4 to form active metabolite, which
is further metabolized to inactive metabolites
Clinical features
 Usually evident by 4 hrs post-ingestion but may be most severe at 8-12hrs with sustained release
preparations
 Mild-moderate CNS effects – nystagmus, dysarthria, ataxia, sedation, delirium, mydriasis,
ophthalmoplegia and myoclonus
 Anticholinergic effects
 Coma requiring intubation and ventilation
 Large overdoses – seizures, hypotension and cardiac conduction abnormalities
 Cardiac dysrhythmias (VT, VF) associated with massive overdoses
Investigations
Screening tests
 ECG, BSL, ethanol and paracetamol level
Specific levels
 Serum Carbamazepine levels –
o Confirms diagnosis
o In severe cases, 6hrly levels provides accurate assessment of patient’s course
 Serial ECG –
o In case of massive overdoses, repeat whenever there is change in level of toxicity, as sodium
channel blockade may become evident later
Carbamazepine level
Clinical features
8-12mg/L
Therapeutic range
>12mg/L
Nystagmus
>20mg/L
CNS system and anticholinergic effects
>40mg/L
Coma, seizures, cardiac conduction abnormalities
Management
Resuscitation, supportive care and monitoring
 Attention to airway, breathing and circulation are paramount
 In c/o ventricular dysrhythmias, sodium bicarbonate boluses to counter sodium channel blockade
 Basic resuscitative measures and good supportive care ensures survival of majority of patients
 Seizures and agitated delirium managed with BDZ
Decontamination
 AC is considered for ingestions <50mg/kg or larger ingestion with controlled release preparations,
which present early and are asymptomatic
 If CNS toxicity evident or impending, AC only administered after airway secured
Enhanced elimination
 MDAC is indicated in severe CBZ poisoning who are intubated, ceased immediately if bowel sounds
disappear
 Extracorporeal elimination useful in severe intoxication. Indicated in c/o prolonged coma with rising
serum levels >48hrs or hemodynamic instability
Disposition and follow up
 Pediatric patients well after 8hrs without sedation or anticholinergic effects may be discharged
 Patients with mild CNS and anticholinergic symptoms are observed in ED for 8hrs and if stable then
transferred to ward
 Patients in coma need ICU management
 Pediatric ingestions with undetectable serum CBZ level after >1hr of ingestion, excluded diagnosis and
may be discharged
Valproic acid
Most valproate overdose result in CNS depression and are managed successfully with good supportive care.
Risk assessment
 Dose-dependent CNS depression may be delayed up to 12hrs
 Increasingly severe multi-system organ effects occur at doses >400mg/kg
 Ingestion >1g/kg is potentially lethal
 Any >200mg/kg dose in children need to be observed in hospital
Toxic mechanism
 Valproate increases levels of GABA and in large dose it interferes with mitochondrial metabolic
pathways
Toxicokinetics
 Usually well-absorbed after oral administration but may be delayed and erratic in overdose
 Peak values may be delayed by up to 18hrs. highly protein bound with Vd of 0.1-0.2L/kg
 Undergoes hepatic metabolism to active metabolites
Dose
Effect
<200mg/kg
Asymptomatic or mild drowsiness and ataxia
200-400mg/kg
Variable CNS depression
400-1000mg/kg
Significant CNS depression. Coma requiring intubation
may be delayed by 12hrs
Multi-organ toxicity with increasing doses
>1000mg/kg
Potential lethal with profound prolonged coma and
multi-organ toxicity – cerebral edema, hypotension,
lactic acidosis, hypoglycemia, hyperammonemia,
Hypernatremia, hypocalcemia and bone marrow
depression
Clinical features
 As above with increasing doses ingested
 Clinical effects parallel rising serum valproate levels
Serum valproate level
Clinical effects
<3500µmol/L
Not usually associated with multi-organ effects
>3500µmol/L
Coma and may have some other organ effects
>7000µmol/L
Life-threatening multi-organ effects
>14000µmol/L
Death expected
Investigations
Screening tests in self-poisoning
 ECG, BSL, paracetamol and ethanol levels
Specific tests
 Serial valproate levels –
o confirm poisoning, refine risk assessment and guide therapy
o repeat levels every 4-6hrs until levels consistently falling
o essential in comatose patients to determine risk of life-threatening multi-organ toxicity and
need for Hemodialysis
 Serial EUC, ABG and FBC – to detect sodium, calcium changes, lactic acidosis and bone marrow
suppression
 Consider valproate overdose in patient with unexplained coma with Hypernatremia, hypocalcemia
and/or lactic acidosis
Management
Resuscitation, supportive care and monitoring
 All patients managed in area equipped with cardiopulmonary monitoring
 Attention for airway, breathing and circulation are paramount
 Intubation anticipated and performed early
 General supportive care measures
Decontamination
 AC not indicated if <400mg/kg ingested
 In larger ingestions, AC given after airway secured
 Repeat dose of AC indicated in 3-4hrs to reduce further absorption and peak serum levels
Enhanced elimination
 Hemodialysis removes valproate efficiently and indicated whenever life-threatening toxicity
suspected
o >1000mg/kg ingestions with serum level >7000µmol/L
o Serum level >10000µmol/L at any time
o Valproate toxicity with lactic acidosis or CVS instability
o Performed before multi-organ toxicity develops
Antidotes – none available
Disposition and follow-up
 Ingestions <200mg/kg observed in ward and fit for discharge once alert and ambulant
 Larger ingestions observed more closely in HDU or ICU
Phenytoin
Phenytoin intoxication results from either repeated supratherapeutic dosing or acute overdose. Intoxication is
usually benign and results in dose-related ataxia and CNS depression.
Risk assessment



Dose-dependent CNS effects: mainly cerebellar occur following acute overdose
Coma and seizures rare even with massive overdoses
Cardiovascular effects not associated with ingestions with any dose
Dose
Effect
10-15mg/kg
Standard therapeutic loading dose
>20mg/kg
Ataxia, dysarthria and nystagmus
>100mg/kg
Potential for Coma and seizures

Children with less than 1-2tablets ingested rarely cause symptoms to need medical assessment and
review.
Toxic mechanism

Phenytoin blocks sodium channels and suppresses membrane post-tetanic potentiation and
hyperexcitability
Toxicokinetics



Absorption slow and erratic after overdose; peak levels may be delayed by 24-48hrs.
Vd 0.6l/kg with high protein binding >90%
Metabolism is by zero-order kinetics and ½ life increases dramatically with small increases in daily
doses

Cytochrome p450 2C9 exhibits genetic polymorphism and inter-individaul variation in elimination
rates are common.
Clinical features








Chronic toxicity presents with gradual onset of ataxia, dysarthria and nystagmus
Mild GI symptoms occurs 2hrs after acute overdose
Onset of neurological toxicity develops slowly over hrs and features include: slow horizontal
nystagmus,dysarthria, ataxia, tremor and vertical nystagmus
Neurological symptoms typically resolve over 2-4days after cessation of drug and as serum levels fall
Coma, seizures and rigidity occur rarely
Hypernatremia and hyperglycemia resulting in HHNS can occur with massive overdose
Permanent neurological cerebellar sequelae have been reported with chronic toxicity
Rapid IV administration of phenytoin (and propylene glycol diluents) is associated with hypotension,
bradycardia, ventricular arrhythmias and asystole.
Investigations
Screening test in c/o overdose
 ECG, BSL, paracetamol and ethanol levels
Specific investigations

Serum phenytoin levels
o
o
Useful to confirm diagnosis
Correlate with clinical toxicity

Nystagmus - >80µmol/L


Severe ataxia - >130-160µmol/L
Coma - >200µmol/L
Management
Resuscitation, supportive care and monitoring




Attention to airway, breathing and circulation are paramount
General supportive measures as indicated
Ambulation under supervision to avoid falls
ECG monitoring not indicated for oral overdoses
Decontamination
 AC for cooperative patient within 4hrs of overdose – reduces toxicity and length of stay
Enhanced elimination


MDAC suggested to enhance elimination but not routinely used
Charcoal hemoperfusion and plasmapharesis used in sever intoxication
Antidotes – none available
Disposition and follow up




Children may be observed at home if no symptoms and in hospital if develop significant ataxia and
drowsiness
Patients with nystagmus, ataxia and drowsiness managed in ward in supportive care
Patients with coma, seizures require airway management and admitted to ICU accordingly
Patients medically fit once able to walk normally
Psychiatric drugs
Anti-psychotic drugs
Clozapine
Deliberate self-poisoning with this atypical antipsychotic agent usually follows a benign course. Care is usually
supportive
Risk assessment
 Poisoning in adults usually follows a benign course
 Coma requiring intubation is rare in pure clozapine ingestions
 Ingestion of single tablet in children can be symptomatic and needs hospitalization. >2.5mg/kg
ingestions may cause sedation, hypersalivation, tachycardia, ataxia and agitation. Extrapyramidal
effects may be seen in following days
Clinical features
 Onset of intoxication rapid, usually within 4 hrs of ingestion
 Lethargy, confusion, sedation, tachycardia and orthostatic hypotension common
 Anticholinergic effects often occur
 Seizures may occur in 5-10% cases
 Extrapyramidal effects common in children
 QT prolongation is uncommon
 Toxicity usually resolves in 24hrs
 Agranulocytosis is seen with therapeutic use of clozapine but has not been reported with overdoses
Investigations
Screening tests in deliberate poisoning
 ECG, BSL, paracetamol and ethanol levels
Specific investigations
 Serial ECGs – ECG at presentation and at 6hrs, cease if QTc normal, continue moitoring if QTc >450ms
 Torsades des pointes not reported
Management
Resuscitation, supportive care and monitoring
 Attention to airway, breathing and circulation are paramount
 Basic resuscitative measures ensure good outcome
 Treat seizures with BDZ
 General supportive measures indicated
 Monitor urine output and treat urinary retention if needed
Decontamination
 Clozapine is rapidly absorbed and toxicity follows a benign course and so AC is not indicated
Enhanced elimination – not useful
Antidotes – not available
Disposition and follow up
 Patients clinically well at 6hrs after ingestion fit for discharge
 Patients with clinical signs need supportive care and monitoring
 Children with h/o ingestion may exhibit extrapyramidal signs for up to a week later
Phenothiazines and butyrophenones
Phenothiazines – chlorpromazine
Butyrophenones – haloperidol, droperidol
Antipsychotic (neuroleptic) agents cause CNS depression, orthostatic hypotension and anticholinergic effects
in overdose. Thioridazine associated with cardiac conduction abnormalities and arrhythmias.
Risk assessment
 Antipsychotics cause dose-dependent CNS depression, tachycardia, hypotension and anticholinergic
effects
 Cardiac toxicity uncommon except with thioridazine
 Dose related effects in overdose
o Chlorpromazine – coma and intubation required at doses >5g
o Thioridazine – coma and cardiac effects >5g
 Seizures uncommon at any dose
 Children – small ingestions can cause significant symptoms and delayed extrapyramidal effects for few
days
Toxic mechanism
 Central dopamine antagonism
 Antagonists at H1, GABA-A, muscarinic, α1 and α2 adrenergic and 5HT receptors
 Cardiac toxicity due to potassium channel blocking effects
Toxicokinetics
 Rapidly absorbed and extensive first pass metabolism at therapeutic doses
 Absorption slow and erratic in overdose
 Lipid-soluble and large volumes of distribution
 Mostly metabolized by CYP450 enzymes to multiple active metabolites which in case of
chlorpromazine can prolonged elimination half-lives
Clinical features
 Onset – usually 2-4hrs after ingestion
 Sedation, ataxia, orthostatic hypotension and tachycardia common
 Fluctuating mental status and intermittent agitated delirium may occur at moderate doses
 Coma may occur with large doses and may last for 18-48hrs
 ECG changes with QRS/QT prolongation common with thioridazine toxic ingestions
 Seizures and EPS uncommon
Investigations
Screening test in case of self-poisoning
 ECG, BSL, paracetamol and ethanol levels
Specific investigations
 Serial ECGs
o Dose dependent QRS/QT prolongation with thioridazine
o Monitor with serial 4-hrly ECG
o Monitor for QT prolongation with other drugs for 12hrs and cease if nil detected
Management
Resuscitation, supportive care and monitoring
 Attention to airway, breathing and circulation paramount
 Treatment of QRS/QT prolongation with IV sodium bicarbonate boluses
 Correct hypoxia, Hypokalemia and administer magnesium sulfate 10mmol over 15mins for torsades
des pointes; start Isoprenaline infusion or overdrive pacing to maintain HR>120/min
 Manage agitated delirium with benzodiazepines
 General supportive care as indicated
Decontamination
 In non-comatose patients – not indicated
 In comatose patients – administer AC after airway protected/intubated NG
Enhanced elimination – not clinically useful
Antidotes – none available
Disposition and follow up
 Patients without mental status or ECG changes after 6hrs of ingestion considered medically fit for
discharge
 Patients with ECG abnormalities monitored for further changes and ceased after QRS/Qt corrected
 Children referred to hospital for any amount ingested and admitted for observation
 Patients with CNS depression may need high level of monitoring dependent on level of toxicity
Antidepressants
SSRIs: Selective Serotonin Re-uptake Inhibitors – citalopram, Escitalopram, Fluoxetine,
Fluvoxamine, Paroxetine, Sertraline
Deliberate self-poisoning with SSRIs is common and usually follows a benign course. Serotonin toxicity occurs
in a minority of patients. Citalopram (escitalopram) appears to be unique in the group with ability to cause
dose-dependent QT interval prolongation.
Risk assessment
 Overdose with SSRIs usually benign irrespective of dose
 Mild symptoms of SS occur in minority (<20%) of patients and usually resolves in less than 12hrs
 Seizures in <4% of patients
 Ingestion of >600mg of citalopram causes QT prolongation but Torsades des pointes is uncommon
 Co-ingestion of other serotonergic agents such as MAOI, SNRIs or tramadol greatly increases the risk
of serotonin syndrome
 Children do not require hospital admission unless symptoms develop, since most ingestions are
benign
Toxic mechanism
 SSRIs enhance central serotonergi neurotransmission by inhibiting re-uptake
 Little affinity for adrenergic, dopaminergic, cholinergic, serotonergic or histamine receptors
Toxicokinetics
 Rapidly absorbed after oral ingestion. Protein bound with large Vd.
 Undergo hepatic metabolism to less active and water soluble metabolites
 Elimination ½ lives of about 24hrs
Clinical features
 Most patients asymptomatic
 Minor symptoms at 4hrs which mostly resolve by 12hrs
 Nausea
 Mild SS manifests as anxiety, tremor, tachycardia, bradycardia and mydriasis
 Seizure uncommon and usually short-lived and respond to BDZ
 Rare reports of Torsades des Pointes and wide-complex bradycardia with citalopram overdose
Investigations
Screening tests in deliberate poisonings
 ECG, BSL, paracetamol and ethanol levels
Specific investigations
 Serial ECG
o Continuous ECG monitoring until 8hrs after significant ingestion or citalopram is required or
until QT interval <440ms, 13hrs if ingestion >1gm.
Management
Resuscitation, supportive care and monitoring
 Attention to airway, breathing and circulation as indicated
 Seizures and agitation managed with BDZ
 Serotonin syndrome managed either supportively with BDZ, oral cyproheptadine or Olanzapine
 Continuous ECG monitoring as dictated by QT measurements
Decontamination
 Alert cooperative patients with >600mg citalopram ingestion AC may be administered, if presentation
within 4hrs
 Other ingestions usually benign and so AC is not indicated
Enhanced elimination – not clinically useful
Antidotes – none available
Disposition and follow up
 Pediatric patients may be observed at home unless they develop symptoms
 All patients with deliberate poisoning should be monitored for at least 12hrs
 Patients with mild SS need supportive care in ward environment for 12-24hrs usually
 Patients with citalopram overdose require monitoring until QTc <440ms
 Patients with severe SS symptoms require management in ICU setting
TCAs: Tricyclic Antidepressants
Amitryptiline, clomipramine, dothiepin, doxepin, imipramine, nortryptiline
TCA poisoing remains a major cause of mortality and morbidity. Deliberate poisoning may lead to rapid onset
of CNS and CVS toxicity.
Risk assessment
 Ingestion of >10mg/kg is potentially life-threatening
 Onset of severe toxicity usually occurs within 2hrs of ingestion
 Seizures and myoclonus more common with dothiepin
Dose
Effect
<5mg/kg
Minimal symptoms
5-10mg/kg
Drowsiness and mild anticholinergic effects
>10mg/kg
Potential for all major effects within 2-4hrs of
ingestion – coma, hypotension, seizures, cardiac
arrhythmias
Anticholinergic effects may be masked by coma
>30mg/kg
Severe toxicity wit pH dependent cardiotoxicity and
prolonged coma

Ingestion of small amounts (1-2tablets) of TCA is potentially lethal, so any ingestion >5mg/kg in
children admitted in hospital for observation
Toxic mechanism
 Noradrealine and serotonin re-uptake inhibitor, GABA-A receptor blocker
 Myocardial toxicity due to blockade of inactivated fast sodium channels
 Blockade of muscarinic, histamine and peripheral post-synaptic α1-adrenergic receptors
 Inhibition of potassium channels and direct myocardial depression
Toxicokinetics
 Rapidly absorbed following oral administration with peak levels within 2hrs
 Highly bound to tissues proteins with large Vd.
 Undergo hepatic metabolism to active metabolites
 Some entero-hepatic circulation occurs
Clinical features
 Severe toxicity is characterized by rapid deterioration in clinical status within 1-2hrs. patients may
present awake and rapidly develop coma, seizures, hypotension and cardiac arrhythmias
 CNS
o Sedation and coma precede CVS signs
o Seizures
o Delirium secondary to anti-cholinergic effects
 Cardiovascular
o Sinus tachycardia with slight rise in BP
o Hypotension due to alpha blocking effects and impaired contractility
o Broad complex tachyarrhythmias
o Broad complex bradycardia pre-arrest
 Anticholinergic effects
o May occur early or may be delayed and prolonged
o Agitation, restlessness and delirium
o Mydriasis
o Dry, warm, flushed skin
o Tachycardia, urinary retention, ileus
o Myoclonic jerks
Investigations
Screening tests in deliberate poisonings
 ECG, BSL, paracetamol and ethanol levels
Specific investigations
 Serial ECG
o Vital tool in management of TCA intoxication
o Diagnostic features include
 Prolongation of PR and QRS intervals
 Large terminal R wave in aVR
 Increased R/S ratio (>0.7) in aVR
 QT prolongation due to potassium channel blockade
o QRS prolongation due to fast sodium channel blockade
o QRS >100ms is predictive of seizures
o QRS >160ms predictive of VT/VF
Management
Resuscitation, supportive care and monitoring
 Acute TCA poisoning ins life threatening and should be managed in a resuscitation bay
 Close clinical and physiological monitoring is mandatory for at least 6hrs since ingestion
 Potential early life-threatening events that may require intervention include:
o Coma
o Respiratory acidosis
o Seizures
o Cardiac arrhythmias
o Cardiac arrest
 At onset of CNS depression (GCS<12) prompt intubation and hyperventilation should occur
 Ventricular dysrhythmias
o Cardioversion and defibrillation unlikely to be successful
o Administer sodium bicarbonate 100meq (1-2meq/kg) q1-2minutes until restoration of
perfusing rhythm
o Lignocaine (1.5mg/kg) – second line therapy if pH>7.5
o Type 1A anti-arrhythmics – Na-channel blockers contraindicated
 Seizures managed with BDZ
 General supportive measures as indicated
 Intubated patients hyperventilated to maintain pH>7.5
Decontamination
 AC not indicated for ingestions <10mg/kg as good outcome is expected with supportive care
 In patients with significant ingestion, AC administered after airway is secured
Enhanced elimination – not clinically useful
Antidotes – sodium bicarbonate for cardiac effects as above
Disposition and follow up
 Patients clinically well with normal ECG, normal mental status, no hypotension or seizures until 6hrs
after ingestion may be considered medically fit for discharge
 Patients with minor abnormalities closely monitored for further deterioration
 Patients requiring intubation and ventilation admitted to ICU
 Resuscitation should continue until patient intubated, boluses of sodium bicarbonate given until
pH>7.5 – good neurologic outcome has been noticed after prolonged arrest and CPR
Venlafaxine
Venlafaxine is a potent SSRI and SNRI. Overdose with this agent is potentially life threatening, frequently
causing seizures and cardiovascular toxicity in large doses. Supportive care and adequate benzodiazepine
sedation usually ensure good outcome.
Risk assessment
 14% of overdose patients have seizures and is dose-dependent
 Onset of seizures may be delayed by about 16hrs with ER preparations – ER preparations are the only
type available now
 Pre-existent seizure disorder increases the risk
 High risk of SS if other serotonergic agents co-ingested
 Massive ingestion >7g associated with cardiovascular effects
Dose
Effect
<1.5g
Risk of seizures <5%
1.5-3g
Risk of seizures 10%
3.1-4.5g
Risk of seizures >30%
4.6-7g
Risk of seizures approaches 100%
Hypotension
Increased QRS and QT duration on 12 lead ECG
>7g
Hypotension and cardiac arrhythmias
 Children – ingestion of 2-3tablets usually does not result in significant toxicity
Clinical features
 Onset of significant features of toxicity may be delayed up to 6-12hrs following ingestion of ER
preparations
 Dysphoria, anxiety, mydriasis, sweating, tremor, clonus, tachycardia (>160/min) and hypotension
common and herald the onset of seizures
 Generalized seizures of short duration and terminated with BDZ
 Severe SS rarely occurs unless co-ingestion of other serotonergic agents present
 Intoxication usually resolves within 24hrs
Investigations
Screening test in deliberate self-poisoning
 ECG, BSL, paracetamol and ethanol levels
Specific investigations
 Serial ECGs
o ECG on presentation and 6hrs post ingestion
o >4.5g ingestions – ECG done every 2hrs and reviewed for QT and QRS prolongation
Management
Resuscitation, supportive care and monitoring
 Life threatening emergency managed in resuscitation bay
 Early intubation and ventilation where ingestion of >7g present
 Urgent life-threatening events which need early management
o Seizures – BDZ
o Broad complex tachycardias – sodium bicarbonate 1-2mmol/kg every 2-3mins
 Increasing agitation, tachycardia and tremor herald onset of seizures and managed empirically with IV
diazepam until gentle sedation achieved
 Hyperthermia may happen as a feature of severe SS and needs early diagnosis, close monitoring and
rapid treatment
 General supportive care measures as indicated
Decontamination

AC in alert and cooperative patients and within 2hrs of ingestion and following >4.5g of ER
preparation
 AC contraindicated in delayed presentation – due to risk of seizures
 >7g – intubate thena dinister AC
Enhanced elimination – not clinically useful
Antidotes – none available
Disposition and follow up
 All patients observed for at least 16-24hrs due to risk of delayed seizures
 Ingestions <4.5g cease cardiac monitoring after 6hrs if ECG normal
 Ingestion >4.5g continue cardiac monitoring until normalization of QRS and QT durations
 Patients with severe SS or cardiac toxicity need intubation and ventilation with ICU care
Cardiovascular drugs
Beta-blockers
Atenolol, Carvedilol, Metoprolol, Propranolol, Sotalol etc.
Propranolol or Sotalol overdose may be life threatening , whereas other β-blocker overdose is usually benign.
Risk assessment:
 The following factors other than dose influence toxicity:
o Ingestion of propranolol or sotalol
o Underlying heart or lung disease
o Co-ingestion of CCB or digoxin
o Advanced age
 Threshold for severe toxicity with propranolol may be as little as 1g.
 Toxicity usually manifests within first few hours unless in case of sotalol and control-release
preparations
 PR interval prolongation is an early sign of toxicity
 All β-blocker ingestions in children need observation especially sotalol and propranolol
Toxic mechanism
 Beta-adrenergic blockade → decreased cAMP and thus decreased chronotropic and inotropic effects
of catecholamines
 Propranolol also has Na-channel blocking actions → QRS widening and ventricular arrhythmias
 Propranolol is also lipid soluble and thus enters CNS causing direct toxicity
 Sotalol also blosks K channels interfering with cardiac repolarization and thus causing QT
prolongation
Clinical features
Occur within 4hrs with onset of β-blockade manifested by a fall in heart rate to around 60bpm.
Cardiovascular:
 Hypotension and bradycardia
 Bradyarrhythmias – sinus bradycardia, first- to third- degree HB, junctional or ventricular bradycardia
 QRS widening with propranolol overdose and predictive of ventricular arrhythmias
 QT prolongation with sotalol overdose
CNS
 Delirium, coma, seizures with propranolol OD
Other
 Bronchospasm, pulmonary edema
 Hyperkalemia
 Hypo/hyperglycemia
Investigations
Bedside
 12-lead ECG and BSL regularly, hourly initially
 Paracetamol level and β-HCG
Specific
 EUC – especially monitor potassium
Management
Resuscitation, supportive care and monitoring

Potentially life-threatening and managed in an area equipped with cardiorespiratory monitoring and
intervention
 Prompt intubation and ventilation early in c/o significant propranolol ingestion, treat with sodium
bicarbonate boluses (as in Na channel blockers).
 Immediate life-threats include:
o Bradycardia and hypotension
 Atropine – 0.01-0.03mg/kg IV temporizing measure
 Adrenaline IV
 Isoprenaline infusion - 4µg/min infusion
 Glucagon 5-10mg bolus, then infusion 1-5mg/hr may be considered
o Wide QRS
 Sodium bicarbonate 1-2meq/kg boluses over 1-2mins
o Torsades des pointes (QT prolongation with sotalol)
 Isoprenaline
 Magnesium
 Overdrive pacing
 Close clinical observation and continuous cardiac monitoring is mandatory
Enhanced elimination – not helpful
Antidotes – Glucagon, high dose insulin-glucose
Disposition
 Asymptomatic patients after 6hrs may be discharged medically
 Patients with ECG changes of any significance should admitted to high dependency ward for
observation
Calcium channel blockers
Verapamil, diltiazem, Amlodipine, nifedipine
Verapamil and Diltiazem commonly cause CV collapse following overdose and this may be delayed by 4-16hrs
if the slow-release preparations are ingested. Other agents do not usually cause severe toxicity.
Risk assessment
 Ingestion of >10 tablets of verapamil SR or diltiazem SR in an adult may cause serious toxicity. All
deliberate self-poisoning are potentially lethal
 Onset of effects usually up to 2hrs with standard preparations and 16hrs or more with SR
preparations
 Co-ingestion of other cardiotoxic medications significantly increases risk of lethality e.g. digoxin, betablockers
 Advanced age and co-morbidities also significantly increase risk
 In children ingestion of 1-2 tablets of SR diltiazem or verapamil is potentially lethal
Toxic mechanism
 CCB prevent opening of L-type calcium channels → decreased calcium influx → vascular muscle
relaxation, slowing of cardiac conduction and reduced force of cardiac contraction → severe
hypotension.
 Diltiazem and verapamil cause central cardiac effects and peripheral vasodilation, whereas the
dihydropyridines cause only vasodilation
 CCB also cause hyperglycemia and lactic acidosis
Clinical features
Following ingestion of standard preparations, onset of symptoms is usually within 1-2hrs. In SR preparation
ingestion, effects may be delayed up to 12-16hrs with peak effects beyond 24hrs.
 Cardiovascular
o Bradycardia, first degree HB and hypotension are early signs
o Progression to refractory shock and death
o Myocardial ischemia, stroke or non-occlusive mesenteric ischemia may occur
 CNS
o Seizures and coma rare
o Coma indicates co-ingestion
 Metabolic
o Hyperglycemia, metabolic and lactic acidosis in severe cases.
Investigations
Bedside
 12-lead ECG and BSL hourly initially
 Paracetamol, ETOH and β-HCG levels
Specific investigations
 12-lead ECG at presentation, at 12-hrs post ingestion; additional ECGs at regular intervals as indicated
 EUC
 Serum calcium level
 Serum lactate and arterial blood gases
 Chest x-ray
 PCWP, CO, systemic resistance monitoring may be required in severe cases
Management
Key objectives
 Early identification of patients at risk
 Initiation of appropriate monitoring
 Consideration of GI decontamination
 Referral to facility capable of advanced resuscitation and intensive care
Resuscitation, supportive care and monitoring
 Life threatening emergency to be managed in area equipped with cardiorespiratory monitoring and
resuscitation
 Hypotension <90mmhg refractory to fluid therapy indicative of significant toxicity and rapid escalation
of therapy indicated
 Life threats include
o Hypotension
o Dysrhythmias
o Cardiac arrest
 Although mentation may be clear until late in toxicity, early intubation and ventilation recommended
for significant ingestions
 Early invasive BP monitoring advised with pulmonary artery measurements to guide inotrope
management
 Approach to hypotension
o Fluid resuscitation – 10-20ml/kg isotonic crystalloid bolus
o Calcium – 0.6-1.0ml/kg of calcium gluconate or 0.2ml/kg of calcium chloride IV over 15mins,
temporizing measure and may be given three times
o Initiate calcium infusion to maintain serum calcium level>2meq/L
o Atropine – 0.6mg x3doses
o Titrate infusion of dopamine, noradrenaline and/or adrenaline
o Insulin dextrose euglycemia
o Sodium bicarbonate 0.5-1.0meq/kg boluses for metabolic acidosis
o Cardiac pacing
 Ventricular pacing to bypass AV block (<60bpm) but does not reverse peripheral
hypoperfusion
o Cardiopulmonary bypass and IABP therapy may be considered as extra-ordinary measures
Decontamination
 Activated charcoal
o Administer PO within 1hr for all patients with standard preparations and <4hrs for SR
preparation ingestions
o Administer to all intubated patients
 Whole bowel irrigation
o Indicated after AC administration if >10tablets of SR drug ingested
Enhanced elimination – not helpful
Antidotes – calcium, atropine, insulin-dextrose euglycemia
Disposition and follow up
 Patients clinically well (vitals and ECG) 4hrs after ingestion of standard preparations and 16hrs after
SR ingestions may be discharged
 All patients with signs of toxicity need intensive care admission for monitoring
Digoxin – Acute overdose
Acute digoxin toxicity manifests as vomiting, hyperkalemia and cardiovascular collapse. It is potentially lethal
and CV complications are refractory to conventional resuscitation measures.
Risk assessment
 Acute intoxication occurs if 10X normal therapeutic dose is ingested
 Potentially lethal doses
o Adults – >10mg
o Children – 4mg
o Serum digoxin level >15nmol/L at any time
o Serum potassium >5.5mmol/L
 Potentially lethal natural glycosides intoxication can occur after ingestion of concoction of tea of
digitalis containing plants or toad skins
Toxic mechanism
 Digoxin inhibits Na-K-ATPase pump → increased intracellular calcium and increased extracellular
potassium → enhanced automaticity and positive inotropic effect
 Digoxin also enhances vagal tone → decreased sinoatrial and atrioventricular node conduction
velocity
Clinical features
Nausea and vomiting are early clinical manifestations of acute digoxin poisoning, developing 2-4hrs post
ingestion. Peak levels occur at 6hrs and death secondary to CV collapse follows at 8-12hrs. specific clinical
features include:
 Gastrointestinal
o Nausea, vomiting and abdominal pain
 Cardiovascular
o Bradycardia
 1st,2nd or 3rd degree heart block
 AF with ventricular response <60bpm
o Increased automaticity
 Ventricular ectopic beats or bigeminy
 SVT with AV block
 VT
o Hypotension
 CNS
o Lethargy, confusion and delirium
Investigations
Bedside
 12 lead ECG at regular intervals, BSL and blood gases
Specific investigations
 Serum digoxin levels
o Confirms poisoning
o Provides indication for antibody treatment
o Perform levels at 4h post ingestion and then q2h for acute ingestions until definitive
treatment or symptoms resolving
 EUC
o Hyperkalemia of any magnitude
High dose insulin euglycemia therapy for beta-blocker and calcium channel blocker overdose
HIET (high-dose insulin euglcemic therapy) was first used to treat verapamil toxicity in humans in 1993, with a
favourable outcome. Since then, in addition to animal studies, there have been about 70 cases reporting the
beneficial use of HIET in humans, with an overall survival rate of 85%. HIET has gained widespread acceptance
as a core therapy for calcium channel blocker toxicity among clinical toxicologists, even though no randomised
controlled trials have been performed to test its efficacy. Unfortunately, awareness of HIET outside of
toxicology circles remains poor, and there is often reluctance to administer the high doses of insulin that are
recommended.
Insulin release is dependent on calcium influx into islet beta cells through L-type calcium channels. This is also
impaired in verapamil toxicity leading to hypoinsulinaemia which together with calcium channel blockerinduced insulin resistance results in hyperglycaemia and a ketoacidotic state.
HIET may allow the heart to overcome the metabolic starvation that results from calcium channel blocker
toxicity, which compounds the direct cardiotoxic effects.
The effects of insulin are numerous:
 increased glucose and lactate uptake by myocardial cells
 improved myocardial function without increased oxygen demand

Increased pyruvate dehydrogenase activity, thus hastening myocardial lactate oxidation and clear the
cytosol of glycolytic by products that can impair calcium handling and cause diastolic dysfunction.
 promotes excitation–contraction coupling and contractility because increased glucose availability
results in:
 increased sarcoplasmic reticulum-associated calcium ATPase activity
 increased cytoplasmic calcium concentrations
 enhanced calcium entrance into mitochondria and sarcolemma
HIET may be best used adjunctively with other measures such as catecholamines, for two reasons:
 Insulin-mediated inotropy is not catecholamine-mediated, and is not affected by β blockers, so
additive effects are likely.
 Although insulin appears to improve myocardial contractility, it has no chronotropic effect and may
cause vasodilation.
Adverse effects of HIET include:
 hypoglycaemia (<3.3 mmol/L in about 16% of cases)
 hypokalaemia
 hypomagnesaemia
 hypophosphataemia
Iron Poisoning
Iron poisoning is characterized by both local gastrointestinal and dose-related systemic toxicity. Toxicity is
determined by the dose of elemental iron ingested.
Risk assessment
 Iron overdose is potentially lethal
 Risk assessment is based upon history of ingested dose and observed evolving clinical features.
 The amount of elemental iron in a ferrous or ferric form is calculated as follows:
o Ferrous sulphate (dried) – dose divided by 3.3
o Ferrous sulphate (heptahydrate) – dose divided by 5
o Ferrous gluconate – dose divided by 9
o Ferrous fumarate – dose divided by 3
o Ferric chloride – dose divided by 3.5
o Ferrous chloride – dose divided by 4
 Confirmation of ingestion by x-ray and interpretation of an iron level at 4-6 hours post-ingestion
 Patients presenting with established systemic iron toxicity have a poor prognosis and may not
respond to medical therapy
 Children – usually trivial, >60mg/kg unlikely.
Dose-related risk assessment for Iron
Elemental Iron dose
<20mg/kg
20-60mg/kg
>60-120mg/kg
>120mg/kg
Effect
Asymptomatic
Gastrointestinal symptoms
Systemic toxicity anticipated
Potentially lethal
Toxic mechanism:
 Local –
o Direct corrosive action on GIT – vomiting diarrhoea to hematemesis and malena
o Local GI fluid losses → significant hypovolemia
o Systemic toxicity does not occur in the absence of GI symptoms
 Systemic –
o Direct cellular toxic effect through unknown mechanisms
o Target organs CVS and liver
o Severe metabolic acidosis due to lactic acid formation and liberation of H+ ions
o Coagulopathy frequently seen
Clinical features
Classically described as five stages, though significant overlap and simultaneity likely
Classical stages of severe iron poisoning
Time post-ingestion
Clinical features
0 – 6 hours
GIT – vomiting, diarrhea and abdominal pain
Fluid losses – hypovolemic shock
6 – 12 hours
Latent period of quiescence – false hope
12 – 48 hours
Disruption of cellular metabolism – shock from:
 Vasodilatation
 Third space losses
 Anion-gap metabolic acidosis
 Hepatorenal failure
2 – 5 days
Acute hepatic failure with jaundice
Coma
Hypoglycaemia
Coagulopathy and raised ALT/AST
Rare phase with high mortality
2 – 6 weeks
Delayed sequelae
 Cirrhotic liver disease
 GI scarring/ strictures
Investigations:
Screening investigations
 BSL
 12-lead ECG
 Paracetamol level
 Β-HCG where indicated
Specific investigations
 Serum iron concentration
o Peak 4 – 6 hours after ingestion. Levels after that unreliable since intracellular distribution
occurs.
o Levels not predictive of outcome but >90µmol/L considered s/o systemic toxicity
o No risk stratification according to levels
 Arterial blood gases – raised anion gap metabolic acidosis
o Serum bicarbonate level a good surrogate marker of toxicity
 Abdominal x-ray – useful for confirming ingestion, planning and monitoring decontamination
Management
Resuscitation, supportive care and monitoring
 Restoration of adequate circulating volume early – boluses of 10-20ml/kg crystalloid
 Ongoing fluid resuscitation
Decontamination
 Activated charcoal DOES NOT ADSORB IRON and thus not indicated
 WBI is the decontamination method of choice for ingestions >60mg/kg and confirmed with x-ray
 Surgical or endoscopic removal may be considered if WBI fails or impractical
Antidotes
 Deferroxamine chelation therapy indicated for systemic toxicity – shock, metabolic acidosis, altered
mental status
 Indication - >90µmol/L at 4-6hrs post ingestion
Desforroxamine therapy
 Cardiac monitoring mandatory during therapy
 Infusion of 15mg/kg/hr, reduce rate if hypotension develops, increasing if tolerated to 40mg/kg/hr for
life-threatening overdoses
 Therapeutic endpoints for infusion – clinical stability and serum iron <60µmol/L
 Avoid infusions for >24hrs
 ADR
o Hypersensitivity
o Hypotension
o ARDS - >24hr infusions
o Toxic retinopathy
o Yersinia sepsis
 Urine may change to classical vin rosé – unreliable sign of success
Disposition and follow up
Children
 <40mg/kg may be managed at home unless symptomatic
 Child who remains asymptomatic after 6hrs may be discharged
Adults
 For ingestions <60mg/kg and asymptomatic after 6hrs – discharge from medical care
 Symptomatic patients and those requiring WBI need HDU admission
 Those with established signs of toxicity and needing DFO therapy need ICU admission
Lead poisoning
Acute lead intoxication is usually due to ingestion. Though rare it is potentially life threatening. Chronic
environmental lead exposure is a major health issue, but is uncommon in Australia.
Risk assessment
 Acute severe lead poisoning can lead to encephalopathy, cerebral edema and death
 Chronic occupational exposure leads to vague multisystem disorder with potential for permanent
sequelae
 Pregnancy – exposure and high lead levels leads to major malformations in children
 Children – neurotoxic leading to intellectual impairment.
Toxic mechanism
 No physiological function
 Interference with intracellular functions – maintenance of cell wall integrity, haem synthesis,
neurotransmitter systems and steroid production
 Major target organs – CNS, kidneys, reproductive and hemopoietic systems
Clinical features
Acute
 Abdominal pain, nausea, vomiting, haemolytic anemia and hepatitis
 Cerebral edema, encephalopathy, seizures and coma pre-terminally
Chronic
 Vague constitutional and multisystem effects
 Impaired concentration, anorexia, vague abdominal pain, emotional lability, weight loss, arthralgia
and impaired coordination
 Subclinical impairment of higher centre function - ↓IQ
Investigations
Screening tests
 BSL
 12-lead ECG
 Paracetamol level
 Β-HCG where indicated
Specific investigations
 Whole blood lead level – most useful
 FBC
o Normochromic, normocytic anemia with basophilic stippling of erythrocytes
 EUC, LFT
 Free erythrocyte protoporphyrin
o Surrogate measure of total body burden of lead when levels >25µg/dL
o FEP is elevated in chronic lead intoxication
 Abdominal x-ray – assists identification of ingested lead foreign bodies
 Nerve conduction and psychomotor testing – objective measure of lead neurotoxicity
Management
Resuscitation, supportive care and monitoring
 Acute resuscitation rarely required
 In case of encephalopathy – management of ABC as indicated
 Administer Mannitol 1g/kg or dexamethasone 10mg if cerebral edema is present
 Seizures treated with benzodiazepines
Decontamination
 Lead foreign body ingestion – options
o Endoscopic retrieval if before duodenum
o If beyond gastro-esophageal junction – high residue diet and PEG to induce diarrhea
o If FB persists after 72 hrs – admit for WBI with PEG
 Shrapnel or bullets – retrieve surgically if possible
Antidotes
 EDTA – IV chelator indicated in acute lead induced encephalopathy or symptomatic patient with level
>100µg/dL
 Succimer (DMSA) is an oral lead chelator used for symptomatic patients without encephalopathy and
asymptomatic patients with levels >60µg/dL for adults and >45µg/dL for children.
Other issues
 Notifiable disease in case of occupational exposure
 Diagnosis of a case should lead to efforts to locate source and screening in all family members and
other potential victims
 Lead levels >10µg/dL should prompt investigations to source and limitation of further exposure
Lithium: acute overdose
Acute lithium OD frequently produces acute GIT symptoms. Provided adequate urinary lithium excretion is
maintained, significant neurotoxicity as in chronic intoxication rarely develops.
Risk assessment
 With normal renal function ingestions <25g unlikely to cause more than minor GIT symptoms
 Acute ingestion of >25g needs supportive care for GIT symptoms, avoiding dehydration, protecting
renal function and sodium depletion
 Renal impairment results in redistribution of lithium and increased potential for delayed neurotoxicity
 Children – most ingestions are non-toxic not requiring hospital management
Toxic mechanism
 Direct irritant to GIT like most metals
 Lithium substitutes for sodium and potassium ions within body modulating second messengers
 May also affect neurotransmitter function
Management
 Only required for patients presenting late with significant intravascular depletion due to GIT losses
and risk of renal dysfunction or
 In patients with established renal dysfunction requiring enhanced elimination with hemodialysis
Lithium: chronic poisoning
Lithium neurotoxicity develops in patients on lithium therapy when renal lithium excretion is impaired for any
reason
Risk assessment
 Consider lithium intoxication in any patient on lithium therapy presenting with neurological
symptoms
 Significant obtundation or seizure activity is indication of severe toxicity and carries risk for
permanent neurological sequelae
 Serum lithium levels correlate poorly with clinical features
Clinical features
 Principally neurological
o Grade 1 (mild)
 Tremor, hyperreflexia, agitation, muscle weakness, ataxia
o Grade 2 (moderate)
 Stupor, rigidity, hypertonia and hypotension
o Grade 3 (severe)
 Coma, convulsions, myoclonus
 GIT symptoms not prominent with chronic toxicity
 Symptoms of underlying disorder causing toxicity may occur. Common precipitants include
o Renal dysfunction
o Diabetes insipidus
o Sodium depletion
o Dehydration
 Complications of long term lithium therapy include –
o Nephrogenic diabetes insipidus
o Hypothyroidism
Investigations
Specific investigations
 Serum lithium level – essential to confirm diagnosis but no correlation with CNS toxicity, but useful in
patient undergoing hemodialysis
 EUC
 Thyroid function tests
Management
Resuscitation, supportive care and monitoring
 Acute resuscitation unlikely necessary except in cases of extreme neurotoxicity with coma and
seizures
 Close correction of fluid and sodium deficit and restoring renal function essential goal
 Cease lithium and renal toxic drugs
Enhanced elimination
 Hemodialysis for any patient with neurological dysfunction and serum lithium level >2.5mmol/L with
renal dysfunction
 Prolonged and repeated sessions of hemodialysis may be needed
Disposition
 Always admit patients with chronic lithium intoxication
 Resolution of neurological deficits may be very slow and take months
Potassium chloride
Deliberate self-poisoning by ingestion of KCl is rare but can be life-threatening causing cardiac arrest. Principal
preparation of concern is sustained release preparations of KCl available in bottles of 100 pills without
prescription.
Risk assessment
 Small ingestion usually benign in patients with normal renal function
 Ingestion of >2.5g may theoretically cause significant toxicity
 Massive ingestions of SR preparations should warrant early referral for hemodialysis
 Patients with renal impairment or cardiac disease at higher risk
 Abdominal x-ray used in risk assessment in case of sustained release preparations which are radioopaque
 Children: ingestion of >3x600mg KCl tablets can cause significant hyperkalemia
Toxic mechanism
 Principal intracellular cation
 Hyperkalemia interferes with electrical conduction in nerve and muscle and when severe causes
cardiac arrest
 Also have direct irritant effect on GIT
Clinical features
 Abdominal pain, nausea and vomiting
 Ileus and mucosal perforation can occur
 K level >6mmol/L – lethargy, confusion, weakness, paraesthesia and hyporeflexia
 K level >8mmol/L – paralysis and cardiac arrest
Investigations
Screening tests
 12-lead ECG, BSL, paracetamol level and β-HCG level when indicated
Specific tests
 Serial 12-lead ECG – peaked T waves (K>5.5mmol/L), PR prolongation, loss of P waves with atrial
paralysis, widening of QRS, Qt prolongation, sine wave appearance and finally asystole
 EUC
 Abdominal x-ray – in c/o large SR preparation ingestion, serial monitoring with x-ray may be required
to monitor decontamination
Management
Resuscitation, supportive care and monitoring
 Attention to ABC as dictated
 Urgent management of initial raised K levels
o Calcium chloride 10ml 10% IV
o Nebulised salbutamol 10-20mg
o Dextrose 50ml 50% with 10 units actrapid insulin IV
o Sodium bicarbonate 50-100meq slow IV
 General supportive measures as required
Decontamination
 Activated charcoal does not bind potassium chloride and is not indicated
 Slow-release KCl preparations are amenable to WBI, but in case of hyperkalemia which has already
set in, hemodialysis should not be delayed
Enhanced elimination
 Hemodialysis is definitive treatment of hyperkalemia after massive potassium overdose and should
occur before cardiac arrest supervenes
 Indications for early hemodialysis decision include:
o Massive OD - >40x600mg KCl tablets confirmed on x-ray
o Renal impairment
o Cardiovascular instability
o Serum K >8.0mmol/L
o Rapidly rising K levels
 Hemodialysis to continue until decontamination of GIT confirmed on x-ray
Disposition and follow up
 All symptomatic patients need close monitoring and HDU/ICU admission for hemodilaysis
Hydrofluoric acid
HF acid exposure may be dermal, inhalation, ocular or oral. Accidental exposure is common. HF ingestion is
potentially lethal.
Risk assessment
 Any dermal exposure may lead to delayed severe pain and tissue injury
 Inhalation injury can lead to pulmonary injury
 Systemic life-threatening fluorosis is associated with ingestion or extensive dermal exposures
o Dermal exposure to 100% HF solution to 2.5% BSA
o Dermal exposure to 70% HF solution to 8% BSA
o Dermal exposure to 23% HF solution to 11% BSA
o Ingestion of >100ml of ≥6% HF solution
o Ingestion of even small quantities of higher concentrations
 Children –any ingestion of HF containing product is potentially lethal
Toxic mechanism
 Fluoride ions bind to calcium and magnesium resulting in cell dysfunction and death
 Systemic toxicity secondary to hypocalcemia, hyperkalemia, hypomagnesemia and acidosis
Clinical features
 Dermal exposure
o Skin contact with <50% HF is not painful and can go unnoticed for hours
o Gradual onset of severe deep unremitting pain without any clinical signs initially
o Pallor and blanching several hours later
o Blistering and tissue loss delayed for hours to days
o Pain usually last 24-36hrs
o Large exposures result in systemic fluorosis
 Inhalational exposure
o Immediate onset of mucosal irritation followed by delayed onset of dyspnoea, cough and
wheeze
o Non-cardiac pulmonary edema in severe cases
 Ingestion
o Low concentration (<20%) minimally corrosive to GIT
o Vomiting, mild throat pain, dysphagia and abdominal pain
o Cardiac arrest from systemic fluorosis can occur at any time after exposure from 30minutes
to 6hrs
 Systemic effects
o Hypocalcemia and hypomagnesemia manifest as tetany and QT prolongation
o Ventricular arrhythmia and cardiac arrest
Investigations
Screening tests
 12-lead ECG, BSL and paracetamol level
Specific investigations
 Serial ECG
o Indicated in all patients with potential systemic fluorosis every 2hrs until monitoring ceases
o Degree of QT prolongation useful marker of hypocalcemia
 Serum or ionised calcium
o Measure at presentation and at 4hrs in all patients with potential systemic HF poisoning
 Endoscopy – once patient stable if risk for corrosive injury
Management
Resuscitation, supportive care and monitoring
 Life threatening emergency in symptomatic patients requiring full cardiopulmonary monitoring
 IV calcium available at bedside
 In case of ventricular dysrhythmias
o
o
Intubate and hyperventilate
Calcium gluconate 10% 60ml or calcium chloride 10% 20mls IV q 5minutes until ROSC. Large
doses may be required
o Sodium bicarbonate 100meq IV
o Magnesium sulphate 10mmol IV
o General supportive measures and analgesia
Decontamination
 Dermal exposure
o Remove clothing
o Irrigate with water
 Ingestion – do not induce vomiting
 Ocular exposure – profuse irrigation with water or saline
Antidotes
 Calcium chloride –calcium gluconate – as above
 In case of severe pain which is refractory – calcium may be administered by subcutaneous infiltration,
regional IV infusion or intra-arterial infusion
Disposition
 Minor skin exposure – outpatient management with calcium gluconate gel
 To return if pain refractory
 Patients requiring calcium infusion need admission until pain settles and controlled with simple
analgesics
 All patients with risk of systemic fluorosis admitted to HDU for monitoring and treatment
Serotonin syndrome versus Neuroleptic malignant syndrome
Serotonin syndrome and neuroleptic malignant syndrome are two drug related emergencies which have
significant symptom overlap and in many cases a clear differentiation between the two may not be possible.
Fortunately the principles of care are similar except for the use of cyproheptadine/olanzapine for SS may
actually worsen NMS, if inappropriately used for misdiagnosis. In cases of unclear diagnosis, avoidance of
these drugs is advisable and ongoing supportive measures may be the best approach to management.
Characteristic
Postulated mechanism
Onset of symptoms
Risk factors
NMS
Dopamine antagonism
Days to weeks
Antipsychotic use,
Poly-pharmacy use,
Rapid increase in dosage
Concurrent use of lithium
Autonomic symptoms
Fever, tachycardia
Diaphoresis
Dilated pupils
BP instability
Constipation, ileus
Muscular rigidity (cogwheel and
leadpipe)
Tremulousness
Underlying psychotic or
Premorbid mood disorder
Leucocytosis,
Elevation of muscle enzymes – CK,
ALT, AST, LDH
Gastrointestinal symptoms
Neurologic symptoms
Psychiatric symptoms
Laboratory findings
Resolution
Complications
Treatment
Drugs that increase serotonin levels
Mechanism
Metabolic serotonin precursor
Inhibit serotonin metabolism
Increase serotonin release
Inhibit serotonin reuptake
Serotonin receptor agonists
Dopamine agonists
5 – 14 days
Aspiration pneumonia, renal
failure, PE, contractures, muscle
weakness
Removal of offending agent
Bromocriptine 2.5mg PO q8h
gradually increased to 5mg q4h
until response
Dantrolene experimentally used
Sedation and paralysis if severe
rigidity
Renal dialysis
SS
Serotonin excess
Minutes to hours
Serotonergic agents
Poly-pharmacy
Concurrent use of lithium
Tramadol, linezolid, amphetamines
MAOIs plus other drugs
Autonomic symptoms similar
Hypotension in severe cases
Nausea, vomiting and diarrhea
Hyperreflexia, myoclonus
Tremulous, clonus
Ataxia ±rigidity less likely
Delirium may be mistaken for
psychosis
Though similar, muscle enzymes
rarely raised unless severely
agitated, aminotransferases rarely
raised
Usually resolves within 24hrs
Falls, seizures, hypotension, DIC,
renal failure and coma early
Removal of offending agent
Benzodiazepines
Serotonin antagonists –
cyproheptadine 4-8mg q1-4h (max
32mg/d)
Olanzapine 10mg SL/IM
experimental
Close monitoring of fluid status
and airway protection
Drug
L-tryptophan
MAOI
Amphetamines, lithium, MDMA
Cocaine, dextromethorphan, TCA, SSRI, venlafaxine
Buspirone, LSD
L-dopa
General proforma to answer toxicology questions
Risk assessment
Toxic mechanism and toxic dose if applicable
Clinical features
Investigations
 Screening – BSL, ETOH, paracetamol, beta-HCG, ECG and ABG where indicated
 Specific – laboratory
o EUC – renal dysfunction
o LFT – esp. in c/o APAP and hepatotoxic drugs
o Coagulation – APAP and anticoagulants
o Drug levels – paracetamol, salicylate, iron, lithium, theophylline, digoxin, carbamazepine,
phenytoin, lead, ethanol, methanol
o ABG – metabolic acidosis, respiratory distress
o CMP
o Serial ECGs
o Serial ABGs
o TFT
 Specific imaging – CXR/AXR/ endoscopic
 EEG/EMG
Management
 Resuscitation, supportive care and monitoring
 Specific
o Decontamination
o Enhanced elimination
o Antidotes
o Ongoing monitoring
o Goals of therapy
 Disposition
o Discharge/ ward/ HDU/ ICU
o Discharge/ monitoring instructions
o Referrals and follow up where indicated
Known lethal doses and ingestions